Закон ампера описывает силу действующую на проводник с током в: Сила Ампера. Сила Лоренца. | Объединение учителей Санкт-Петербурга

Содержание

Сила Лоренца (эл. И магн. Часть). Закон Ампера. Действие магнитного поля на движущийся заряд. Сила Лоренца

Электрический ток – это совокупность упорядоченно движущихся заряженных частиц. Поэтому действие магнитного поля на проводник с током есть результат действия поля на движущиеся заряженные частицы внутри проводника.

Силу, действующую на движущуюся заряженную частицу со стороны магнитного поля, называют силой Лоренца.

Модуль силы Лоренца равен отношению модуля силы Ампера, действующей на участок проводника, к числу заряженных частиц в этом участке проводника:

Сила Ампера равна , сила тока равна(см. стр. 12). Подставив эти выражения в формулу для силы Лоренца, получим:

где — угол между векторами скорости и магнитной индукции.

Направление силы Лоренца определяют для положительного заряда по правилу левой руки. (Для отрицательного заряда сила Лоренца будет направлена в противоположную сторону).

Так как сила Лоренца перпендикулярна скорости частицы, то она не совершает работу. А, согласно теореме о кинетической энергии, это означает, что сила Лоренца не меняет кинетическую энергию частицы и, следовательно модуль ее скорости. Под действием силы Лоренца меняется лишь направление скорости частицы.

Закон Ампера

Поместим в магнитное поле проводник длинной l, по которому течет ток I. На проводник действует сила, прямо пропорциональная силе тока, текущего по проводнику, индукции магнитного поля, длине проводника, и зависящая от ориентации проводника в магнитном поле. |F|=IBlsin, где угол между направлением тока в проводнике и направлением вектора магнитной индукции B, Направление силы Ампера определяется по

правилу левой руки: если левую руку расположить так, что магнитные силовые линии входят в ладонь, четыре вытянутых пальца направить по току, то отогнутый большой палец укажет направление силы. Очевидно, что сила Ампера равна нулю, если проводник расположен вдоль силовых линий поля и максимальна, если проводник перпендикулярен силовым линиям. Движение заряженных частиц в магнитном поле. На проводник с током в магнитном поле действует сила Ампера FА IBlsin.Ток, в свою очередь, это направленное движение заряженных частиц. Сила тока равна I=qnvS, где q – заряд частицы, n-концентрация движущихся заряженных частиц, v-средняя скорость их направленного движения, S-площадь поперечного сечения проводника. Подставив I в выражение для FА, получим FА= qnvSBlsin, где nsl=N – общее число частиц, создающих ток. Тогда сила, действующая на отдельный движущийся заряд – сила Лоренца, равна Fл=qvB
sin
. где — угол между векторами скорости и магнитной индукции. Направление силы Лоренца определяется для положительно заряженной частицы по правилу левой руки.

Закон Ампера

Сила, действующая на проводник с током в магнитном поле, называется силой Ампера.

Экспериментальное изучение магнитного взаимодействия показывает, что модуль силы Ампера пропорционален длине проводника с током, силе тока и зависит от ориентации проводника в магнитном поле.

Опыт показывает, что магнитное поле, вектор индукции которого направлен вдоль проводника с током, не оказывает влияния на ток. Поэтому модуль силы зависит лишь от модуля составляющей вектора магнитной индукции, перпендикулярной проводнику.

Закон Ампера заключается в следующем. Сила Ампера равна произведению магнитной индукции поля на силу тока, длину участка проводника и на синус угла между магнитной индукцией и участком проводника:

Направление силы ампера определяется по правилу левой руки: если левую руку расположить так, чтобы перпендикулярная составляющая магнитной индукции входила в ладонь, а четыре вытянутых пальца были направлены по направлению тока, то отогнутый на 90 большой палец покажет направление силы, действующей на отрезок проводника.

Магнитное взаимодействие проводников с током используется в Международной системе для определения единицы сила тока –ампера.

Ампер –сила неизменяющегося тока, который при прохождении по двум параллельным прямолинейным проводникам бесконечной длины и ничтожно малого кругового сечения, расположенным на расстоянии 1 м друг от друга в вакууме, вызывал бы между этими проводниками силу магнитного взаимодействия, равную Н на каждый метр длины.

Закон ампера презентація

Закон ампера презентація

Скачать закон ампера презентація rtf

26-11-2021

Дана презентація є супроводжуючою до уроку за продуктивною технологією «Сила Ампера». в ній висвітлено всі етапи уроку. Дана презентація — спеціально організоване спілкування з аудиторією, мета якого переконати або спонукати її до певних дій. Презентація на урок Фізика скачати.  Сила Ампера. Урок з фізики в 11 класі Рівень стандарту Методист комунальної установи “Талалаївський районний центр з обслуговування закладів освіти” Дівенко В.Г. Слайд № 2. Сила Ампера. Действие магнитного поля на проводник с током. Сила, действующая на проводник с током в магнитном поле, называется силой Ампера. Сила действия однородного магнитного поля на проводник с током прямо пропорциональна силе тока, караван у пустелі 4 клас образотворче мистецтво проводника, модулю вектора индукции магнитного поля, синусу угла между вектором индукции магнитного поля и проводником: F=B.I.ℓ. sin α — закон Ампера.

Сила Ампера. ФИО автора: Лоскутова Людмила Михайловна –преподаватель физики.  Тема: Магнитное поле. Постоянные магниты. Сила Ампера. Методическая тема (цель): подготовка конкурентоспособных специалистов, владеющих общими и профессиональными компетенциями. Цель урока. Модуль вектора магнитной индукции. Сила Ампера. Не удалось посмотреть файл. Не удалось загрузить страницу.  Закон Ампера. Правило левой руки. Действие магнитного поля на рамку с током. Применение силы Ампера. Не удалось посмотреть файл. Не удалось загрузить страницу.

Сила Ампера Сила Ампера – сила, действующая на проводник с током, помещенный в магнитное поле. F. — презентация. Презентация была опубликована 8 лет назад izotermacenter.ru Получить код презентации. Скачать. Копировать в буфер обмена.  Скачать бесплатно презентацию на тему «Закон Ампера. Сила Ампера Сила Ампера – сила, действующая на проводник с током, помещенный в магнитное поле. F.» в izotermacenter.ru (PowerPoint). Похожие презентации. Закон Ампера — если провод, по которому течет ток, находится в магнитном поле, то на каждый из носителей тока действует сила Ампера. Закон Ампера в векторной форме. Закон Ампера устанавливает, что на проводник с током, помещенный в однородное магнитное поле, индукция которого В, действует сила, пропорциональная силе тока и индукции магнитного поля.

Ампер открыл взаимодействие электрических токов и установил закон этого взаимодействия (закон Ампера). 3 слайд. Описание слайда: Гипотеза Ампера Согласно гипотезе Ампере внутри молекул вещества циркулируют элементарные электрические токи.  Сила Ампера – это сила, с которой магнитное поле действует на проводник с током Опыт. 5 слайд. Описание слайда.

Опыты Ампера ( г.) французский физик А. Ампер наблюдал силовое взаимодействие двух проводников с токами и установил закон взаимодействия токов. 6 слайд. Описание слайда: Ампер Андре Мари ( – ) Один из основоположников электродинамики. Открыл механическое взаимодействие токов и установил закон этого взаимодействия (закон Ампера). Построил первую теорию магнетизма. 7 слайд. Описание слайда.

прощения, что ампера гдз граматика англійська мова 6 клас закон Достаточно спорно, поспорил автором… Вебмастер читатели играют

Закон Ампера. На проводник с током со стороны магнитного поля действует сила Ампера, направление которой определяется по правилу левой руки. Предлагается презентация урока физики в 11 классе «Сила Ампера». Объем и сложность материала соответствуют базовому курсу физики при двух часах в неделю. Урок содержит блиц — вопросы для повторения, самостоятельную работу (2 варианта, длительность 5 минут) по теме «Магнитное поле», объяснение новой темы и задания на закрепление изученного материала.

Найбільший вибір готових шкільних презентацій! Андре-Марі Ампер (варіант 2) — презентація з фізики на порталі GDZ4YOU — з нами вчитись дійсно легко, відчуй це!  Андре-Марі Ампер Народився 20 січня в м. Ліоні. З юнацького віку займався математикою та природознавством. Був спершу репетитором в Політехнічній школі. Содержание презентации: План: 1. Закон Ампера. Сила взаимодействия параллельных токов 2. Контур с током в магнитном поле 3. Эффект Холла 4. Движение заряженных частиц в постоянном магнитном поле. Скачать презентацию бесплатно.ру. Здесь Вы можете бесплатно скачать презентацию, поделиться своей и создать презентацию online. Меню.

Закон Ампера — закон взаимодействия постоянных токов. Установлен Андре Мари Ампером в Из закона Ампера следует, что параллельные проводники с постоянными токами, текущими в одном направлении, притягиваются, а в противоположных — отталкиваются. Законом Ампера называется также закон, определяющий силу, с которой магнитное поле действует на малый отрезок проводника с током. Начиная с года. Кафедра физики ЗАКОН АМПЕРА Сила презентація двух параллельных токов Ток I 1 создает вокруг себя магнитное поле, линии магнитной индукции которого B1 концентрические окружности. I1 dF1 R I2 Направление линий магнитной индукции. определяется правилом правого винта, модуль вектора B1 равен 0 I 1 B1 2 R Направление силы dF1, с которой поле B1 действует на элемент dl. тока I 2, определяется из закона Ампера dF I dl, B. Общая физика. «Магнитостатика». 8.

Вы можете изучить и скачать доклад-презентацию на тему Закон Ампера. Презентация на заданную тему содержит 10 слайдов. Для просмотра воспользуйтесь проигрывателем, если материал оказался полезным для Вас — поделитесь повсякденне життя 19 століття презентація с друзьями с помощью социальных кнопок и добавьте наш сайт презентаций в закладки! Презентации» Образование» Закон Ампера. Слайды и текст этой презентации. Слайд 1. Описание слайда. Закон Ампера гласит: Сила взаимодействия двух параллельных проводников пропорциональна произведению величин токов в проводниках, пропорциональна длине этих проводников и обратно пропорциональна расстоянию между. – сила взаимодействия двух параллельных проводников  Еще одно открытие Ампера – это закон действия магнитного поля на проводник с током. Он выражается прежде всего в действии магнитного поля на виток или рамку с током.

Вы можете изучить и скачать доклад-презентацию на тему Закон Ампера. Презентация на заданную тему содержит 10 слайдов. Для просмотра воспользуйтесь проигрывателем, если материал оказался полезным для Вас — поделитесь им с амперами с помощью социальных кнопок и добавьте наш сайт презентаций в закладки! Презентации» Образование» Закон Ампера. Слайды и текст этой презентации. Слайд 1. Описание слайда.

попался! Неоднократно презентація закон ампера могу сейчас поучаствовать обсуждении

Закон Ампера». Перейти к файлу. Заказать учебную работу. Чтобы посмотреть презентацию с картинками, оформлением и слайдами, скачайте ее файл и откройте в PowerPoint на своем компьютере. Текстовое содержимое слайдов презентации. § 5. Применение закона Ампера. Громкоговоритель. Зная направление и модуль силы, действующей на любой участок проводника с током, можно вычислить суммарную силу, действующую на весь замкнутый проводник. Для этого надо найти сумму сил, действующих на каждый участок проводника с током. Закон Ампера используют для расчета сил, действующих на проводники с током, во многих технических устройствах.

Текстовое содержимое слайдов презентации: СИЛА АМПЕРА и её применение Автор: Терехова Галина Анатольевна. МБОУ лицей№12 г. Химки, Московской обл. Сила Ампера Сила, с которой магнитное поле действует на проводник, с током помещённый в это конспект уроку з української літератури 6 клас леся українка співець. Величину этой силы определяют с помощью закона Ампера. Сила Ампера. Принцип действия электродвигателя. КГКП «Алматинский многопрофильный колледж».  СИЛА АМПЕРААндре-Мари Ампер — французский физик, математик и естествоиспытатель. Номер слайда 7. izotermacenter.ru?v=U0svq.

Андре-Мари Ампер – гг. В году Ампером был установлен закон, определяющий силу, действующую на отдельный участок проводника. — + — + Слайд 5. Текст слайда: Андре-Мари Ампер – гг. Великий французский физик и математик, один из основоположников электродинамики. Ввел в физику понятие. Модуль вектора магнитной индукции. Сила Ампера. Не удалось посмотреть файл. Не удалось загрузить страницу.  Закон Ампера. Правило левой руки. Действие магнитного поля на рамку с током. Применение силы Ампера. Не удалось посмотреть файл. Не удалось загрузить страницу.

Опыты Ампера ( г.) французский физик А. Ампер наблюдал силовое взаимодействие двух проводников с токами и установил закон взаимодействия токов. 6 слайд. Описание слайда: Ампер Андре Мари ( – ) Один из основоположников электродинамики. Открыл механическое взаимодействие токов и установил закон этого взаимодействия (закон Ампера). Построил первую теорию магнетизма. 7 слайд. Описание слайда. Опыты Ампера. Задача была peшенa Ампером. Ампер открыл закон взаимодействия токов, решив тем самым проблему магнитного взаимодействия. Закон взаимодействия полюсов магнитов, который Кулон считал фундаментальным, оказался следствием из закона, открытого Ампером. «Все в совокупности, — писал об Ампере Максвелл, — и теория и эксперимент как будто появились в полной зрелости и полном вооружении из головы Ньютона электричества.

2.Сила Ампера имеет направление в пространстве, которое определяется по правилу левой руки: если левую руку расположить так, чтобы вектор магнитной индукции входил в ладонь, а вытянутые пальцы были направлены вдоль тока, то отведенный большой палец укажет направление действия силы. Слайд 4. Применение силы Ампера. Слайд 5. Применение силы Ампера. Ориентирующее действие МП на контур с током.

Закон Ампера. презентация к уроку физики (11 класс) по теме. Опубликовано — — Анисимова Марта Александровна. Презентация к уроку физики. Скачать  Сила Ампера действующая на проводник с током, находящийся в МП зависит от: Значения силы тока в проводнике; Длины проводника; Интенсивности МП; Угла, образованного вектором магнитной индукции и проводником (максимального значения достигает, когда вектор магнитной индукции перпендикулярен проводнику).

Сила Ампера Сила Ампера – сила, действующая на проводник с током, помещенный в магнитное поле. F. — презентация. Презентация была опубликована 8 лет назад izotermacenter.ru Получить код презентации. Скачать. Копировать в буфер обмена.  Скачать бесплатно презентацию на тему «Закон Ампера. Сила Ампера Сила Ампера – сила, действующая на проводник с током, помещенный в магнитное поле. F.» в izotermacenter.ru (PowerPoint). Похожие презентации.

топик Интересно, ампера презентація закон забавное сообщение конечно, прошу прощения, это мне

Андре-Мари Ампер – гг. В году Ампером был установлен закон, определяющий силу, действующую на отдельный участок проводника. — + — + Слайд 5. Текст слайда: Андре-Мари Ампер – гг. Великий французский физик и математик, один из основоположников электродинамики. Ввел в физику понятие. Презентация для школьников на тему «Закон Ампера» по физике. izotermacenter.ru — удобный каталог с возможностью скачать powerpoint презентацию бесплатно.  Аннотация к презентации. Презентация для школьников на тему «Закон Ампера» по физике. izotermacenter.ru — удобный каталог с возможностью скачать powerpoint презентацию бесплатно. Формат. pptx (powerpoint).

Закон Ампера гласит: Сила взаимодействия двух параллельных проводников пропорциональна произведению величин токов в проводниках, пропорциональна длине этих проводников и обратно пропорциональна расстоянию между. – сила взаимодействия двух параллельных проводников  Еще одно открытие Ампера – это закон действия магнитного поля на проводник с током. Он выражается прежде всего в действии магнитного поля на виток или рамку с током. Сила Ампера Сила Ампера – сила, действующая на проводник с током, помещенный в магнитное поле. F. — презентация. Презентация была опубликована 8 лет назад izotermacenter.ru Получить код презентации. Скачать. Копировать в буфер обмена.  Скачать бесплатно презентацию на тему «Закон Ампера. Сила Ампера Сила Ампера – сила, действующая на проводник с током, помещенный в магнитное поле. F.» в izotermacenter.ru (PowerPoint). Похожие презентации.

Установлен Андре Мари Ампером в Из закона Ампера следует, что параллельные проводники с постоянными токами, текущими в одном направлении, притягиваются, а в противоположных — отталкиваются. Законом Ампера называется также закон, определяющий силу, с которой магнитное поле действует на малый отрезок проводника с током. Сила, с которой магнитное.

Сила Ампера. Учитель Кононов Геннадий Григорьевич. СОШ № Приморский район.  Закон Ампера Андре-Мари Ампер ( – ) Основные труды в области электродинамики. Автор первой теории магнетизма. Предложил правило для определения нап-равления действия магнитного поля на магнитную стрелку (правило Ампера). Открыл взаимодействие токов и устано-вил закон этого взаимодействия (закон Ампера).

Презентация для школьников на тему «Закон Ампера» по физике. izotermacenter.ru — удобный каталог с возможностью скачать powerpoint презентацию бесплатно.  Аннотация к презентации. Презентация для школьников на тему «Закон Ампера» по физике. izotermacenter.ru — удобный каталог с возможностью скачать powerpoint презентацию бесплатно. Формат. pptx (powerpoint).

Реферат физика — сила Лоренца, закон Лоренца и Ампера

ТАШКЕНТСКИЙ УНИВЕРСИТЕТ ИНФОРМАЦИОННЫХ ТЕХНОЛОГИЙ ИМЕНИ МУХАММАД АЛЬ-ХОРАЗМИЙ

ТЕМА: Сила Лоренца. Законы Лоренца и Ампера.

Студента группы: 076-20 Выполнил:Вайменов Серказы Проверил: _________________

ПЛАН:

1.


7

2.Закон Ампера. 7

Закон Ампера — один из важнейших и полезнейших законов в электротехнике, без которого немыслим научно-технический прогресс. Этот закон был впервые сформулирован в 1820 году Андре Мари Ампером. Из него следует, что два расположенные параллельно проводника, по которым проходит электрический ток, притягиваются, если направления токов совпадают, а если ток течёт в противоположных направлениях, то проводники отталкиваются. Взаимодействие здесь происходит посредством магнитного поля, которое перманентно возникает при движении заряженных частиц. Математически закон Ампера в простой форме выглядит так: 7

7

где F — это сила Ампера (сила, с которой проводники отталкиваются или притягиваются), где B — магнитная индукция; I — сила тока; L — длина проводника; α — угол между направлением тока и направлением магнитной индукции. 7

2.https://pue8.ru/elektrotekhnik/404-zakon-ampera-opredelenie-formula.html 10

Введение

А. Ампер в 1820 г. опытным путем установил законы силового взаимодействия рамок с токами.

В конце XIX в. Х.А. Лоренц обобщением опытных данных получил выражение для силы, действующей на электрический заряд, движущийся в магнитном поле.

Направление линий магнитной индукции связано с направлением тока в проводнике. Направление силовых линий магнитного поля, создаваемого проводником с током, определяется по правилу буравчика (если правовинтовой буравчик ввинчивать по направлению тока, то направление вращения рукоятки буравчика совпадет м направлением линий магнитной индукции).

Одним из проявлений магнитного поля является его силовое воздействие на движущиеся электрические заряды и проводники с током. В 1820г. А. Ампером был установлен закон, определяющий силу, действующую на элемент тока в магнитном поле. Так как создать обособленный элемент нельзя, то Ампер изучал поведение подвижных проволочных замкнутых контуров различной формы. Им было установлено, что на проводник с током помещенный в однородное магнитное поле индукции В, действует сила, пропорциональная длине отрезка проводника L, силе тока I, протекающего по проводнику, и индукции магнитного поля В. Впоследствии этот вывод получил название закона Ампера. Используя закон Ампера, можно вычислить силу, действующую на проводник с током в магнитном поле.

Движущиеся электрические заряды создают вокруг себя магнитные поля, которые распространяются в вакууме со скоростью света с. Если же заряд движется во внешнем магнитном поле, то происходит силовое взаимодействие магнитных полей, определяемое по закону Ампера. Процесс взаимодействия магнитных полей исследовался Лоренцем, который вывел формулу для расчета силы действующей со стороны магнитного поля на движущуюся заряженную частицу. Данная сила получила название силы Лоренца.
1.Сила Лоренца. Так как электрический ток представляет собой упорядоченное движение зарядов, то действие магнитного поля на проводник с током есть результат его действия на отдельные движущиеся заряды.

Силу, действующую со стороны магнитного поля на движущиеся в нем заряды, называют силой Лоренца.

Из закона Ампера (1) следует, что сила Лоренца определяется соотношением:

где q-величина движущегося заряда;
V-модуль его скорости;
B — модуль вектора индукции магнитного поля;
ɑ- угол между вектором скорости заряда и вектором магнитной индукции.

Рис. 1.

Направление вектора Fл определяется по правилу левой руки:

четыре пальца по направлению скорости движения положительного заряда V;

перпендикулярная скорости составляющая вектора индукции входит в ладонь;

отогнутый большой палец дает направление силы Лоренца Fл (см. рис. 1).

в векторной форме

Если частица имеет отрицательный заряд, то направление силы Лоренца противоположное.

Поскольку сила Лоренца направлена под углом 90° к скорости движения заряженной частицы в каждой точке траектории, то работа силы Лоренца при движении заряженной частицы в магнитном поле равна нулю: Согласно теореме о кинетической энергии, изменение кинетической энергии этой заряженной частицы
.
Следовательно, = const, т.е. кинетическая энергия частицы, движущейся в магнитном поле, не изменяется, а значит, заряженная частица в магнитном поле движется с постоянной по модулю скоростью, а направление скорости изменяется непрерывно.

Действие силы Лоренца наблюдается и в природе, и во многих технических устройствах. Например, сила Лоренца отклоняет заряженные частицы, вторгающиеся из космоса и попадающие в магнитное поле Земли, к полярным областям, где они вызывают полярные сияния. Действие магнитного поля на движущиеся заряженные частицы используется для управления движением электронов в телевизионных трубках, в ускорителях и т.д.

В технике сила Лоренца используется очень часто: во всех двигателях и генераторах именно она приводит во вращение ротор под действием электромагнитного поля статора. Таким образом, в любых электромоторах и электроприводах основным видом силы является Лоренцева. Кроме того, она применяется в ускорителях заряженных частиц, а также в электронных пушках, которые раньше устанавливались в ламповых телевизорах. В кинескопе испускаемые пушкой электроны отклоняются под влиянием электромагнитного поля, что происходит при участии Лоренцевой силы. Кроме того, эта сила используется в масс-спектрометрии и масс-электрографии для приборов, способных сортировать заряженные частицы в зависимости от их удельного заряда (отношение заряда к массе частицы). Это позволяет с высокой точностью определять массу частиц. Также находит применение в других КИП, например, в бесконтактном способе измерения расхода электропроводящих жидких сред (расходомеры). Это очень актуально, если жидкая среда обладает очень высокой температурой (расплав металлов, стекла и др.).
Сила Лоренца очень похожа по своему принципу на силу Ампера, разница заключается в том, что последняя действует на весь проводник, который в целом электрически нейтральный, а сила Лоренца описывает влияние электромагнитного поля лишь на единичный движущийся заряд.  

2.Закон Ампера.

Закон Ампера — один из важнейших и полезнейших законов в электротехнике, без которого немыслим научно-технический прогресс. Этот закон был впервые сформулирован в 1820 году Андре Мари Ампером. Из него следует, что два расположенные параллельно проводника, по которым проходит электрический ток, притягиваются, если направления токов совпадают, а если ток течёт в противоположных направлениях, то проводники отталкиваются. Взаимодействие здесь происходит посредством магнитного поля, которое перманентно возникает при движении заряженных частиц. Математически закон Ампера в простой форме выглядит так:

где F — это сила Ампера (сила, с которой проводники отталкиваются или притягиваются), где B — магнитная индукция; I — сила тока; L — длина проводника; α — угол между направлением тока и направлением магнитной индукции.

Любые узлы в электротехнике, где под действием электромагнитного поля происходит движение каких-либо элементов, используют закон Ампера. Самый широко распространённый и используемый чуть-ли не во всех технических конструкциях агрегат, в основе своей работы использующий закон Ампера — это электродвигатель, либо, что конструктивно почти то же самое, генератор.

Закон Ампера для проводника произвольной ормы в неоднородном магнитном поле:

 

 Idl — элемент тока, малый участок проводника, имеющий направление, совпадающее с направлением тока в проводнике.

Направление силы Ампера определяется по правилу левой руки: если 4 пальца левой руки расположить по направлению тока в проводнике, а индукция магнитного поля входит в ладонь, то отогнутый большой палец укажет направлнение силы Ампера.

 Опыт Ампера: взаимодействие двух бесконечных параллельных проводников с током


 токи в одном направлении — проводники притягиваются



токи в противоположных направлениях — проводники отталкиваются



Взаимодействие двух прямолинейных проводников с током

Если считать, что проводник 2 находится в магнотном поле, созданном током в проводнике 1 (и наоборот), то индукция магнитного поля

По закону Ампера

где — расстояние между проводниками, Δl — элемент длины проводника 2.

Именно под действием силы Ампера происходит вращение ротора, поскольку на его обмотку влияет магнитное поле статора, приводя в движение. Любые транспортные средства на электротяге для приведения во вращение валов, на которых находятся колёса, используют силу Ампера (трамваи, электрокары, электропоезда и др). Также магнитное поле приводит в движение механизмы электрозапоров (электро двери, раздвигающиеся ворота, двери лифта). Другими словами, любые устройства, которые работают на электричестве и имеющие вращающиеся узлы основаны на эксплуатации закона Ампера. Также он находит применение во многих других видах электротехники, например, в громкоговорителях. В громкоговорителе или динамике для возбуждения мембраны, которая формирует звуковые колебания используется постоянный магнит. На него под действием электромагнитного поля, создаваемого расположенным рядом проводником с током, действует сила Ампера, которая изменяется в соответствии с нужной звуковой частотой.

Вывод

Используемая литература


  1. К.Т. Суяров, – I глава «Магнитное поле» Физика 11кл 2018г, 15-19с.

  2. Мякишев, Г.Я. Физика 10кл 1995г, 187-191с

Интернет сайты


  1. https://light-fizika.ru/index.php/11-klass?layout=edit&id=127

  2. https://pue8.ru/elektrotekhnik/404-zakon-ampera-opredelenie-formula.html

  3. https://infourok.ru/referat-magnitnoe-pole-fizika-10-klass-4065003.html

  4. http://www.allbest.ru/

Правило левой руки




Проводник с током в магнитном поле. Магнитная индукция.


Если проводник, по которому проходит электрический ток, внести в магнитное поле, то в результате взаимодействия магнитного поля и проводника с током проводник будет перемещаться в ту или иную сторону.
Направление перемещения проводника зависит от направления тока в нем и от направления магнитных линий поля.

Допустим, что в магнитном поле магнита NS находится проводник, расположенный перпендикулярно плоскости рисунка; по проводнику протекает ток в направлении от нас за плоскость рисунка.

Ток, идущий от плоскости рисунка к наблюдателю, обозначается условно точкой, а ток, направляющийся за плоскость рисунка от наблюдателя,— крестом.


Движение проводника с током в магнитном поле
1 — магнитное поле полюсов и тока проводника,
2 — результирующее магнитное поле.

Всегда всё уходящее на изображениях обозначается крестом,
а направленное на смотрящего — точкой.

Под действием тока вокруг проводника образуется свое магнитное поле рис.1.
Применяя правило буравчика, легко убедиться, что в рассматриваемом нами случае направление магнитных линий этого поля совпадает с направлением движения часовой стрелки.

При взаимодействии магнитного поля магнита и поля, созданного током, образуется результирующее магнитное поле, изображенное на рис.2.
Густота магнитных линий результирующего поля с обеих сторон проводника различна. Справа от проводника магнитные поля, имея одинаковое направление, складываются, а слева, будучи направленными встречно, частично взаимно уничтожаются.

Следовательно, на проводник будет действовать сила, большая справа и меньшая слева. Под действием большей силы проводник будет перемещаться по направлению силы F.

Перемена направления тока в проводнике изменит направление магнитных линий вокруг него, вследствие чего изменится и направление перемещения проводника.

Для определения направления движения проводника в магнитном поле можно пользоваться правилом левой руки, которое формулируется следующим образом:

Если расположить левую руку так, чтобы магнитные линии пронизывали ладонь, а вытянутые четыре пальца указывали направление тока в проводнике, то отогнутый большой палец укажет направление движения проводника.

Сила, действующая на проводник с током в магнитном поле, зависит как от тока в проводнике, так и от интенсивности магнитного поля.

Основной величиной, характеризующей интенсивность магнитного поля, является магнитная индукция В. Единицей измерения магнитной индукции является тесла (Тл=Вс/м2).

О магнитной индукции можно судить по силе действия магнитного поля на проводник с током, помещенный в это поле. Если на проводник длиной 1 м и с током 1 А, расположенный перпендикулярно магнитным линиям в равномерном магнитном поле, действует сила в 1 Н (ньютон), то магнитная индукция такого поля равна 1 Тл (тесла).

Магнитная индукция является векторной величиной, ее направление совпадает с направлением магнитных линий, причем в каждой точке поля вектор магнитной индукции направлен по касательной к магнитной линии.

Сила F, действующая на проводник с током в магнитном поле, пропорциональна магнитной индукции В, току в проводнике I и длине проводника l, т. е.
F=BIl.

Эта формула верна лишь в том случае, когда проводник с током расположен перпендикулярно магнитным линиям равномерного магнитного поля.
Если проводник с током находится в магнитном поле под каким-либо углом а по отношению к магнитным линиям, то сила равна:
F=BIl sin a.
Если проводник расположить вдоль магнитных линий, то сила F станет равной нулю, так как а=0.
(Подробно и доходчиво в видеокурсе «В мир электричества — как в первый раз!»)


Тема «Действие магнитного поля на движущийся заряд. Сила Лоренца»

Просмотр содержимого документа
«Тема «Действие магнитного поля на движущийся заряд. Сила Лоренца»»

Тема «Действие магнитного поля на движущийся заряд. Сила Лоренца» »

11 класс

Вопросы на повторение

  • Что называют линиями магнитной индукции?
  • Закон Ампера?
  • Правило левой руки для определения направления силы Ампера.
  • В каких единица выражается магнитная индукция?

Задача на повторение

Сила Лоренца

Определение: Силу, действующую на движущуюся заряженную частицу со стороны магнитного поля, называют  силой Лоренца  

В честь великого голландского

физика X. Лоренца (1853— 1928).

Модуль силы Лоренца

равен отношению модуля силы F, действующей на участок проводника длиной Δl, к числу N заряженных частиц, упорядоченно движущихся в этом участке проводника:

Сила Лоренца

  • Уравнение для силы тока в проводнике:

Сила Лоренца

, где

Сила Лоренца

Сила Лоренца перпендикулярна векторам  магнитной индукции и скорости заряженной частицы.

Правило Левой Руки

Если левую руку расположить так, чтобы линии магнитной индукции входили в ладонь, а четыре вытянутых пальца были направлены по движению положительно заряженной частицы, то отогнутый на 90 градусов большой палец укажет направление действующей на проводник силы

Направление действующей силы

Траектория движения частиц

1.Если

2. Если ⊥ , то частица описывает траекторию в виде окружности

3. Если не⊥ , то частица описывает траекторию в виде винтовой линии

Радиус кривизны

Согласно второму закону Ньютона:

Отсюда радиус:

Период оборота

Время, за которое частица делает полный оборот (период обращения), равно:

Решение задач

Решение задач

Вопросы на повторение

1. Чему равен модуль силы Лоренца?

2. Как движется заряженная частица в однородном магнитном поле, если начальная скорость частицы перпендикулярна линиям магнитной индукции?

3. Как определить направление силы Лоренца?

Домашнее задание

§ 6, Упражнение 1

Рымкевич:

Самостоятельная работа

11 класс

Сила Ампера. Сила Лоренца (дидактический материал)

1 вариант

1. Пря­мо­ли­ней­ный про­вод­ник дли­ной 0,2 м на­хо­дит­ся в од­но­род­ном маг­нит­ном поле с ин­дук­ци­ей 4 Тл и рас­по­ло­жен под углом 30° век­то­ру ин­дук­ции. Чему равен мо­дуль силы, дей­ству­ю­щей на про­вод­ник со сто­ро­ны маг­нит­но­го поля при силе тока в нем 2 А?

 

2.  Пря­мо­ли­ней­ный про­вод­ник дли­ной L с током I по­ме­щен в од­но­род­ное маг­нит­ное поле так, что на­прав­ле­ние век­то­ра маг­нит­ной ин­дук­ции B пер­пен­ди­ку­ляр­но про­вод­ни­ку. Если силу тока умень­шить в 2 раза, а ин­дук­цию маг­нит­но­го поля уве­ли­чить в 4 раза, то дей­ству­ю­щая на про­вод­ник сила Ам­пе­ра

 

3.

Про­тон p, вле­тев­ший в зазор между по­лю­са­ми элек­тро­маг­ни­та, имеет ско­рость  V, пер­пен­ди­ку­ляр­но век­то­ру ин­дук­ции B маг­нит­но­го поля, на­прав­лен­но­му вер­ти­каль­но. Куда на­прав­ле­на дей­ству­ю­щая на про­тон сила Ло­рен­ца F?

 

1) от на­блю­да­те­ля

2) к на­блю­да­те­лю

3) го­ри­зон­таль­но впра­во

4) вер­ти­каль­но вниз

4. Про­вод­ник с силой тока 10 А дли­ной 2 м на­хо­дит­ся в од­но­род­ном маг­нит­ном поле с ин­дук­ци­ей 0,5 Тл. При­чем на­прав­ле­ние маг­нит­но­го поля со­став­ля­ет 30° с на­прав­ле­ни­ем тока. Чему равна сила со сто­ро­ны маг­нит­но­го поля, дей­ству­ю­щая на про­вод­ник?

5. От­ри­ца­тель­ный то­чеч­ный заряд q дви­жет­ся со ско­ро­стью  V в од­но­род­ном маг­нит­ном поле с ин­дук­ци­ей В так, как по­ка­за­но на ри­сун­ке. На каком из сле­ду­ю­щих ри­сун­ков пра­виль­но по­ка­за­но на­прав­ле­ние силы Ло­рен­ца F , дей­ству­ю­щей на заряд со сто­ро­ны маг­нит­но­го поля?

 

6.

 К пря­мо­ли­ней­но­му вер­ти­каль­но­му участ­ку про­во­да, по ко­то­ро­му про­те­ка­ет по­сто­ян­ный ток I, мед­лен­но под­нес­ли спра­ва по­сто­ян­ный маг­нит, как по­ка­за­но на ри­сун­ке. Куда на­прав­ле­на маг­нит­ная сила, дей­ству­ю­щая на про­вод?

 

1) впра­во → 2) влево ←

3) «на нас»  4) от нас

7. Элек­трон вле­та­ет в од­но­род­ное маг­нит­ное поле с ин­дук­ци­ей 5 Тл со ско­ро­стью 1 км/с, на­прав­лен­ной под не­ко­то­рым углом 60° к си­ло­вым ли­ни­ям маг­нит­но­го поля. Най­ди­те зна­че­ния мо­ду­ля силы Ло­рен­ца, дей­ству­ю­щей на элек­трон. Заряд электрона q = — 1,6 *10-19 Кл.

2 вариант

1. На уча­сток пря­мо­го про­вод­ни­ка дли­ной 50 см в од­но­род­ном маг­нит­ном поле с ин­дук­ци­ей 2 Тл при силе тока в про­вод­ни­ке 20 А и на­прав­ле­нии век­то­ра ин­дук­ции маг­нит­но­го поля под углом 30° к про­вод­ни­ку дей­ству­ет сила Ам­пе­ра, при­бли­зи­тель­но рав­ная

2.  Пря­мо­ли­ней­ный про­вод­ник дли­ной L с током I по­ме­щен в од­но­род­ное маг­нит­ное поле пер­пен­ди­ку­ляр­но ли­ни­ям ин­дук­ции B. Как из­ме­нит­ся сила Ам­пе­ра, дей­ству­ю­щая на про­вод­ник, если его длину уве­ли­чить в 2 раза, а силу тока в про­вод­ни­ке умень­шить в 4 раза?

 

3.

Про­тон р вле­та­ет по го­ри­зон­та­ли со ско­ро­стью у в вер­ти­каль­ное маг­нит­ное поле ин­дук­ци­ей В между по­лю­са­ми элек­тро­маг­ни­та (см. ри­су­нок). Куда на­прав­ле­на дей­ству­ю­щая на про­тон сила Ло­рен­ца ?

1) вер­ти­каль­но вниз 

2) вер­ти­каль­но вверх 

3) го­ри­зон­таль­но к нам 

4) го­ри­зон­таль­но от нас 

4. Пря­мо­ли­ней­ный про­вод­ник дли­ной 0,5 м, по ко­то­ро­му течет ток 6 А, на­хо­дит­ся в од­но­род­ном маг­нит­ном поле. Мо­дуль век­то­ра маг­нит­ной ин­дук­ции 0,2 Тл, про­вод­ник рас­по­ло­жен под углом 30° к век­то­ру В. Сила, дей­ству­ю­щая на про­вод­ник со сто­ро­ны маг­нит­но­го поля, равна

 

5.   От­ри­ца­тель­ный то­чеч­ный заряд q дви­жет­ся со ско­ро­стью V в од­но­род­ном маг­нит­ном поле с ин­дук­ци­ей В так, как по­ка­за­но на ри­сун­ке. На каком из сле­ду­ю­щих ри­сун­ков пра­виль­но по­ка­за­но на­прав­ле­ние силы Ло­рен­ца F , дей­ству­ю­щей на заряд со сто­ро­ны маг­нит­но­го поля?

 

 6.

К пря­мо­ли­ней­но­му го­ри­зон­таль­но­му участ­ку про­во­да, по ко­то­ро­му про­те­ка­ет по­сто­ян­ный ток I, мед­лен­но под­нес­ли снизу по­сто­ян­ный маг­нит, как по­ка­за­но на ри­сун­ке. Куда на­прав­ле­на маг­нит­ная сила, дей­ству­ю­щая на про­вод?

 

1) вверх ↑ 2) вниз ↓

3) «на нас»  4) «от нас»  

7. Заряженная частица электрон влетает в однородное магнитное поле с индукцией 2 Тл в вакууме со скоростью 105 м/с перпендикулярно линиям магнитной индукции. Вычислим силу, действующую на электрон. Заряд электрона q = — 1,6 *10-19 Кл.

Задачи на применение правила левой руки

Идёт приём заявок

Подать заявку

Для учеников 1-11 классов и дошкольников

Описание презентации по отдельным слайдам:

ПО ТЕМЕ : «МАГНИТНОЕ ПОЛЕ. ПРИМЕНЕНИЕ ПРАВИЛ БУРАВЧИКА, ПРАВОЙ И ЛЕВОЙ РУКИ» Урок решения задач

Цели урока: 1. развитие интереса, умения и навыков к решению тестовых и графических задач. 2. совершенствование полученных знаний и умений 3. уметь решать задачи на описание магнитного поля тока и его действия :сила Ампера и сила Лоренца направление линий магнитной индукции.

Определить направление силы Ампера: N S FA

Определить направление силы Ампера: N S FA

Определить направление силы Ампера: N S FA

Определить направление силы Ампера: N S FA

Применяя правило левой руки, определи направление силы, с которой магнитное поле будет действовать на проводник с током. Предполагаемые направления силы Ампера указаны стрелочками. 1 2 3 4 а) 1, б)2, в)3, г)4

Применяя правило левой руки, определи направление силы, с которой магнитное поле будет действовать на проводник с током. Предполагаемые направления силы Ампера указаны стрелочками. 1 2 3 4 а) 1, б)2, в)3, г)4

Применяя правило левой руки, определи направление силы, с которой магнитное поле будет действовать на проводник с током. Предполагаемые направления силы Ампера указаны стрелочками. 1 2 3 4 а) 1, б) 2, в) 3, г) 4

Обнаружить магнитное поле можно по. А) по действию на любой проводник, Б) действию на проводник, по которому течет электрический ток, В) заряженный теннисный шарик, подвешенный на тонкой нерастяжимой нити, Г) на движущиеся электрические заряды. а) А и Б, б) А и В, в) Б и В, г) Б и Г.

Закончить фразу: «Если электрический заряд неподвижен, то вокруг него существует. а) магнитное поле, б) электрическое поле, в) электрическое и магнитное поле.

Закончить фразу: «Если электрический заряд движется, то вокруг него существует. а) магнитное поле, б) электрическое поле, в) электрическое и магнитное поле.

Закончить фразу: «Вокруг проводника с током существует. а) магнитное поле, б) электрическое поле, в) электрическое и магнитное поле.

Какие силы проявляются во взаимодействии двух проводников с током? а) силы магнитного поля, б) силы электрического поля, в) сила всемирного тяготения.

Какие утверждения являются верными? А.В природе существуют электрические заряды. Б.В природе существуют магнитные заряды. В.В природе не существует электрических зарядов. Г.В природе не существует магнитных зарядов. а) А и Б, б) А и В, в) А и Г, г) Б, В и Г.

На рисунке показана картина магнитных линий прямого тока. В какой точке магнитное поле самое сильное? а) б) в)

Два параллельных проводника, по которым текут токи противоположных направлений. а) взаимно притягиваются, б) взаимно отталкиваются, в) никак не взаимодействуют.

Определить направление тока по известному направлению магнитных линий

Определить направление тока в проводнике по направлению магнитных линий

Список литературы Учебник для общеобразовательных учебных заведений – Физика 9 класс, Перышки А.В. и Гутник Е.М. «Сборник задач по физике» (В.И. Лукашик, Е.В. Иванова) «Физика». Краткий справочник школьника. «Физика». Большой справочник для школьников и поступающих в вузы. «Физика». Словарь школьника. «Большой справочник школьника». «Учебный справочник школьника».

  • Филатова Анна ФанузовнаНаписать 13859 14.10.2017

Номер материала: ДБ-752806

Не нашли то что искали?

Вам будут интересны эти курсы:

Все материалы, размещенные на сайте, созданы авторами сайта либо размещены пользователями сайта и представлены на сайте исключительно для ознакомления. Авторские права на материалы принадлежат их законным авторам. Частичное или полное копирование материалов сайта без письменного разрешения администрации сайта запрещено! Мнение редакции может не совпадать с точкой зрения авторов.

Ответственность за разрешение любых спорных моментов, касающихся самих материалов и их содержания, берут на себя пользователи, разместившие материал на сайте. Однако редакция сайта готова оказать всяческую поддержку в решении любых вопросов связанных с работой и содержанием сайта. Если Вы заметили, что на данном сайте незаконно используются материалы, сообщите об этом администрации сайта через форму обратной связи.

Осталось ждать: 20 сек.

Установите безопасный браузер

Предпросмотр документа

Какова индукция магнитного поля, в котором на проводник с длиной активной части 5 см действует сила 50 мН? Сила тока в проводнике 25 А. Проводник расположен перпендикулярно магнитным линиям.

На прямой проводник длиной 0,5 м, расположенный перпендикулярно магнитным линям поля с индукцией 0,02 Тл, действует сила 0,15 Н. Определите силу тока, протекающего по проводнику.

Определите силу, действующую со стороны магнитного поля с индукцией 0,1 Тл на протон, движущийся со скоростью 3*106 м/с, если угол между направлениями скорости и линий магнитной индукции 900. Заряд протона 1,6 *10-19 Кл.

Определите направление тока в проводнике, находящемся в магнитном поле.

Стрелка указывает направление действующей на проводник силы.

50742114587690right32082552994113018115231018224379Укажите стрелками направление линий магнитного поля, если известно, что проводник с током под действием магнитного поля отклоняется вправо. Укажите

северн6ый и южный магнитные полюса.

В каком направлении действует сила на проводник, расположенный

перпендикулярно к плоскости чертежа, если ток в проводнике идет

Презентация была опубликована 6 лет назад пользователемrusedu.ru

Похожие презентации

Презентация по предмету «Право» на тему: «Итоговый урок 10 Решение задач По теме : «Магнитное поле. Применение правил буравчика, правой и левой руки» Выполнила: учитель Удомельской СОШ4 Сергеева.». Скачать бесплатно и без регистрации. — Транскрипт:

1 Итоговый урок 10 Решение задач По теме : «Магнитное поле. Применение правил буравчика, правой и левой руки» Выполнила: учитель Удомельской СОШ4 Сергеева В.Е.

2 Цели урока: 1. развитие интереса, умения и навыков к решению тестовых и графических задач. 2. совершенствование полученных знаний и умений 3. умение решать задачи на описание магнитного поля тока и его действия :сила Ампера и сила Лоренца направление линий магнитной индукции.

3 Определить направление силы Ампера: N S FAFA

6 Определить направление силы Ампера: N S FAFA

7 Как изменится сила Ампера, действующая на прямолинейный проводник с током в однородном магнитном поле при увеличении индукции в 3 раза? Проводник расположен перпендикулярно вектору индукции. а) уменьшится в 9 раз; б) уменьшится в 3 раза; в) увеличится в 3 раза; г) увеличится в 9 раз

8 Как изменится сила Ампера, действующая на прямолинейный проводник с током в однородном магнитном поле, при увеличении силы тока в проводнике в 2 раза? Проводник расположен перпендикулярно вектору индукции. а) уменьшится в 2 раза; б) уменьшится в 4 раза; в) увеличится в 2 раза; г) увеличится в 4 раза

9 Проводник с током помещен в магнитное поле с индукцией В. По проводнику течет ток I. Как изменится модуль силы Ампера, если положение проводника относительно магнитных линий изменяется – сначала проводник был расположен параллельно линиям индукции, потом его расположили под углом 30 0 к линиям индукции, а потом его расположили перпендикулярно линиям индукции. а) модуль силы Ампера возрастал, б) модуль силы Ампера убывал, в) модуль силы Ампера оставался неизменным в течение всего процесса.

10 Как изменится сила Ампера, действующая на прямолинейный проводник с током в однородном магнитном поле, при увеличении индукции магнитного поля в 3 раза и увеличении силы тока в 3 раза? Проводник расположен перпендикулярно вектору индукции. а) уменьшится в 9 раз; б) уменьшится в 3 раза; в) увеличится в 3 раза; г) увеличится в 9 раз.

11 Применяя правило левой руки, определи направление силы, с которой магнитное поле будет действовать на проводник с током. Предполагаемые направления силы Ампера указаны стрелочками а) 1, б)2, в)3, г)4

12 Применяя правило левой руки, определи направление силы, с которой магнитное поле будет действовать на проводник с током. Предполагаемые направления силы Ампера указаны стрелочками а) 1, б)2, в)3, г)4

13 Применяя правило левой руки, определи направление силы, с которой магнитное поле будет действовать на проводник с током. Предполагаемые направления силы Ампера указаны стрелочками а) вверх, б)вниз, в) к нам, г) от нас.

14 Применяя правило левой руки, определи направление силы, с которой магнитное поле будет действовать на проводник с током. Предполагаемые направления силы Ампера указаны стрелочками а) 1, б) 2, в) 3, г) 4

15 Определить положение полюсов магнита, создающего магнитное поле. а) слева – северный полюс, б) слева – южный полюс.

16 Обнаружить магнитное поле можно по. А) по действию на любой проводник, Б) действию на проводник, по которому течет электрический ток, В) заряженный теннисный шарик, подвешенный на тонкой нерастяжимой нити, Г) на движущиеся электрические заряды. а) А и Б, б) А и В, в) Б и В, г) Б и Г.

17 Закончить фразу: «Если электрический заряд неподвижен, то вокруг него существует. а) магнитное поле; б) электрическое поле; в) электрическое и магнитное поле;

18 Закончить фразу: «Если электрический заряд движется, то вокруг него существует. а) магнитное поле, б) электрическое поле, в) электрическое и магнитное поле.

19 Закончить фразу: «Вокруг проводника с током существует. а) магнитное поле, б) электрическое поле, в) электрическое и магнитное поле.

20 Какие силы проявляются во взаимодействии двух проводников с током? а) силы магнитного поля, б) силы электрического поля, в) сила всемирного тяготения.

21 Какие утверждения являются верными? А.В природе существуют электрические заряды. Б.В природе существуют магнитные заряды. В.В природе не существует электрических зарядов. Г.В природе не существует магнитных зарядов. а) А и Б, б) А и В, в) А и Г, г) Б, В и Г.

22 На рисунке показана картина магнитных линий прямого тока. В какой точке магнитное поле самое сильное? а) б) в)

23 Два параллельных проводника, по которым текут токи противоположных направлений. а) взаимно притягиваются, б) взаимно отталкиваются, в) никак не взаимодействуют.

24 Два параллельных проводника длиной по 1 м, расположенные на расстоянии 1 м друг от друга при протекании по ним электрического тока, притягиваются с силой Н. Это значит, что по проводникам текут токи. а) противоположных направлений по 1 А, б) одного направления по 1 А, в) противоположных направлений по 0,5 А, г) одного направления по 0,5 А.

25 Как будут взаимодействовать друг с другом два параллельных проводника А и Б? а) они будут притягиваться, б) они будут отталкиваться, в) они не будут взаимодействовать.

26 Определить направление тока по известному направлению магнитных линий

27 Определить направление тока в проводнике по направлению магнитных линий

28 Магнитная стрелка отклонится, если её разместить вблизи. А) вблизи потока электронов, Б) вблизи потока атомов водорода, В) вблизи потока отрицательных ионов, Г) вблизи потока положительных ионов, Д) вблизи потока ядер атома кислорода. а) все ответы верны, б) А, Б, В, и Г, в) Б, В, Г, г) Б, В, Г, Д

29 Определите силу Лоренца,действующую на электрон

30 Определите силу Лоренца,действующую на протон

31 Список литературы Учебник для общеобразовательных учебных заведений – Физика 9 класс, Перышки А.В. и Гутник Е.М. «Сборник задач по физике» (В.И. Лукашик, Е.В. Иванова) «Физика». Краткий справочник школьника. «Физика». Большой справочник для школьников и поступающих в вузы. «Физика». Словарь школьника. «Большой справочник школьника». «Учебный справочник школьника».

ЗАКОН AMPERES

ЗАКОН AMPERES

Магнитное поле на расстоянии r от очень длинного прямого провода, несущего установившийся ток I, имеет величину, равную

(31.)

и направление, перпендикулярное r и I. Интеграл по путям по окружности с центром вокруг проволоки (см. рисунок 31.1) равно

(31,2)

Здесь мы использовали тот факт, что магнитное поле является касательным в любой точке на круговой траектории интегрирования.

Рисунок 31.1. Магнитное поле, создаваемое током. Любой произвольный путь можно рассматривать как набор радиальных сегменты (r изменяется, а [theta] остается постоянным) и круглые сегменты ([тета] изменяется, а r остается постоянным). Для радиальных сегментов магнитная поле будет перпендикулярно перемещению и продукту скейлера между магнитное поле и смещение равны нулю. Рассмотрим теперь небольшой циркуляр отрезок траектории вокруг провода (см. рисунок 31.2). Интеграл по путям на этом круговом отрезке равно

(31.3)

Рисунок 31.2. Интеграл по небольшому круговому пути. Уравнение (31.3) показывает, что вклад этого циркуляра отрезок к общему интегралу по путям не зависит от расстояния r и только зависит от изменения угла [Дельта] [тета]. Для замкнутого пути общее изменение угла составит 2 пи, и уравнение (31.3) можно переписать как

(31.4)

Это выражение — Закон Ампера :

«Интеграл от B вокруг любого замкнутого математического пути равен u 0 раз больше тока, перехваченного областью, охватывающей путь «

Пример: Задача 31,5

Шесть параллельных алюминиевых проволок небольшого, но конечного радиуса лежат в тот же самолет. Провода разделены равным расстоянием d, и они несут равные токи I в том же направлении.Найдите магнитное поле в центре первый провод. Предположим, что токи в каждом проводе распределены равномерно. по его поперечному сечению.

Схематическое изображение проблемы показано на рисунке 31.3. Магнитное поле генерируется одиночным проводом равно

(31,5)

где r — расстояние от центра провода. Уравнение (31.5) имеет вид правильно для всех точек за пределами провода, и поэтому может использоваться для определения магнитное поле, создаваемое проводами 2, 3, 4, 5 и 6.Поле на центр провода 1 из-за тока, протекающего в проводе 1, может быть определен с помощью Закон Ампера и равен нулю. Суммарное магнитное поле в центре провод 1 можно найти путем векторного сложения вкладов каждого из шести провода. Поскольку направление каждого из этих вкладов одинаково, полное магнитное поле в центре провода 1 равно

(31.6)

Рисунок 31.3. Проблема 31.5

Соленоид — это устройство, используемое для создания однородного магнитного поля.Оно может быть изготовленным из тонкой проводящей проволоки, намотанной в спиральную спиральную катушку, состоящую из множества витков. Магнитное поле внутри соленоида можно определить, суммируя магнитные полей, порождаемых N отдельными кольцами (где N — количество витков соленоид). Мы ограничимся обсуждением магнитного поля, создаваемого соленоид к тому, который генерируется идеальным соленоидом, который бесконечно длинный, и имеет очень плотно намотанные катушки.

Идеальный соленоид обладает поступательной и вращательной симметрией.Однако, поскольку силовые линии магнитного поля должны образовывать замкнутые контуры, магнитное поле не может быть направлен в радиальном направлении (в противном случае будут созданы силовые линии или разрушен на центральной оси соленоида). Таким образом, мы заключаем, что силовые линии в соленоиде должны быть параллельны оси соленоида. Величина магнитного поля можно получить, применив закон Ампера.

Рисунок 31.4. Идеальный соленоид. Рассмотрим путь интеграции, показанный на рисунке 31.4. Путь интеграл магнитного поля вокруг этого пути интегрирования равен

(31,7)

где L — горизонтальная длина пути интегрирования. Текущий вложенный по пути интегрирования равен N . I 0 где N — число витков на пути интегрирования, а I 0 — это ток в каждом витке соленоида. Используя закон Ампера, заключаем, что

(31.8)

или

(31,9)

где n — количество витков соленоида на единицу длины. Уравнение (31.9) показывает, что магнитное поле B не зависит от положения внутри соленоид. Мы заключаем, что магнитное поле внутри идеального соленоида равно униформа.

Пример: проблема 31.14

По длинному соленоиду n витков на единицу длины проходит ток I, и по длинной прямой проволоке, идущей вдоль оси этого соленоида, проходит ток I ‘.Найдите суммарное магнитное поле внутри соленоида на расстоянии r от оси. Опишите форму силовых линий магнитного поля.

Магнитное поле, создаваемое соленоидом, однородное, направленное параллельно оси соленоида, и имеет величину

(31.10)

Магнитное поле длинного прямого провода, по которому течет ток I ‘, имеет величина равна

(31.11)

и направлена ​​перпендикулярно направлению r и I ‘.Направление Таким образом, провод B перпендикулярен направлению В соль . Чистое магнитное поле внутри соленоида равно векторная сумма B провода и B sol . Его величина равна на номер

(31.12)

Угол a между направлением магнитного поля и осью z равен по

(31,13)

Пример: проблема 31.15

Коаксиальный кабель состоит из длинного цилиндрического медного провода радиусом r 1 окружена цилиндрической оболочкой с внутренним радиусом r 2 и внешний радиус r 3 (см. рисунок 31.5). Проволока и оболочка несут равные и противоположные токи I равномерно распределены по их объемам. Находить формулы для магнитного поля в каждой из областей r 1 , r 1 2 , r 2 3 и r> r 3 .

Силовые линии магнитного поля представляют собой окружности с центром на оси симметрии магнитного поля. коаксиальный кабель. Сначала рассмотрим путь интегрирования с r 1 . Интеграл по путям B на этом пути равен

. (31,14)

Ток, заключенный в этом пути интеграции, равен

. (31,15)

Применяя закон Фарадея, мы можем связать вложенный ток с интегралом по путям из Б

(31.16)

Следовательно, магнитное поле B равно

. (31.17)

Рисунок 31.5. Проблема 31.15. В области между проводом и оболочкой закрытые ток равен I, а интеграл по путям магнитного поля равен уравнение (31.14). Закон Ампера гласит, что

(31.18)

а магнитное поле равно

(31,19)

В третьей области (r 2 3 ) интеграл по путям магнитного поля по круговой траектории радиуса r определяется выражением экв.(31,14). Включенный ток равен

(31.20)

Следовательно, магнитное поле равно

. (31.21)

Ток на пути интегрирования с радиусом r> r 3 равна нулю (поскольку ток в проводе и в оболочке течет в противоположные направления). Следовательно, магнитное поле в этой области также равняется нулю.

Магнитная сила, действующая на частицу с зарядом q, движущуюся со скоростью v, равна равно

(31.22)

Эта сила всегда перпендикулярна направлению движения частицы, и поэтому изменит только направление движения, а не величину скорости. Если заряженная частица движется в однородной магнитной поле с напряженностью B, перпендикулярное скорости v, то величина магнитной силы определяется как

(31,23)

и его направление перпендикулярно v. В результате этой силы частица будет совершать равномерное круговое движение.Радиус круга равен определяется требованием, чтобы сила магнитного поля была равна центростремительная сила. Таким образом

(31,24)

Радиус орбиты r равен

. (31,25)

где p — импульс заряженной частицы. Расстояние, пройденное частица за один оборот равна

(31,26)

Время T, необходимое для совершения одного оборота, равно

. (31.27)

Частота этого движения равна

. (31,28)

и называется циклотронной частотой . Уравнение (31.28) показывает что циклотронная частота не зависит от энергии частицы, и зависит только от его массы m и заряда q.

Влияние магнитного поля на движение заряженной частицы можно использовать для определения некоторых его свойств. Одним из примеров является измерение заряд электрона.Электрон, движущийся в однородном магнитном поле, будет описал круговое движение с радиусом, задаваемым уравнением (31.25). Предположим, что электрон ускоряется потенциалом V 0 . Конечная кинетическая энергия электрона равно

(31.29)

Импульс электрона p определяется его кинетической энергией

(31.30)

Таким образом, радиус кривизны траектории электрона равен

. (31.31)

Уравнение (31.31) показывает, что измерение r можно использовать для определения отношение массы к заряду электрона.

Другое приложение влияния магнитного поля на движение заряженная частица — циклотрон. Циклотрон состоит из вакуумированного резонатора. помещен между полюсами большого электромагнита. Полость разрезается на две части D-образные детали (называемые деэ) с промежутком между ними. Колеблющийся высокий напряжение подается на пластины, создавая колеблющееся электрическое поле в область между двумя деями.Заряженная частица, инжектированная в центр циклотрон будет совершать равномерное круговое движение в течение первой половины один оборот. Частота движения частицы зависит от ее массы, ее заряд и напряженность магнитного поля. Частота генератора выбирается таким образом, что каждый раз, когда частица пересекает зазор между деями, она будет ускоряться электрическим полем. По мере увеличения энергии иона его радиус кривизны будет увеличиваться, пока не достигнет края циклотрон и извлекается.При движении в циклотроне ион будет многократно пересекать пропасть между деками, и она будет ускорена до максимума энергии.

До сих пор мы предполагали, что направление движения заряженного частица перпендикулярна направлению магнитного поля. Если это в этом случае произойдет равномерное круговое движение. Если направление движения ион не перпендикулярен магнитному полю, это приведет к спиральному движению. Скорость заряженной частицы можно разложить на две составляющие: одну параллельно и один перпендикулярно магнитному полю.Магнитная сила действующее на частицу, будет определяться составляющей ее скорости перпендикулярно магнитному полю. Проекция движения частица на плоскости x-y (предполагается, что она перпендикулярна магнитному полю) будет круглым. Магнитное поле не повлияет на компонент движение параллельно полю, и эта составляющая скорости останется постоянный. Конечным результатом будет спиральное движение.

Заряженная частица, движущаяся в области с электрическим и магнитным полем, будет испытать общую силу равную

(31.32)

Эта сила называется силой Лоренца .

Рисунок 31.6. Заряженная частица движется в скрещенных E и B поля. Рассмотрим частный случай, когда электрическое поле перпендикулярно магнитному полю. Движение заряженной частицы в таком регион может быть достаточно сложным. Заряженная частица с положительным зарядом q и скорость v движется в этом поле (см. рисунок 31.6). Направление частица, показанная на рисунке 31.6 перпендикулярна как электрическому полю, так и магнитное поле. Электрическая сила, действующая на частицу, направлена по направлению электрического поля и имеет величину, равную

(31,33)

Магнитная сила, действующая на заряженную частицу, направлена ​​перпендикулярно к как v, так и B, и имеет величину, равную

. (31,34)

Суммарная сила, действующая на частицу, складывается из этих двух компонентов и имеет величина, равная

(31.35)

Если заряженная частица имеет скорость, равную

(31,36)

тогда результирующая сила будет равна нулю, и движение частицы будет быть равномерным линейным движением. Устройство со скрещенными электрическим и магнитным полями называется селектором скорости. Если прорезь размещена в соответствующем положения, он будет переносить только те частицы, скорость которых определена величинами электрического и магнитного полей.

Рисунок 31.7. Ток в магнитном поле. Метод, используемый для определения плотности и знака заряда. носителей в металле основана на силах, действующих на скрещенные поля E и B на носители заряда. На схеме, показанной на рисунке 31.7, показана металлическая полоса. проводящий ток в указанном направлении и помещенный в однородный магнитный поле с направлением магнитного поля, перпендикулярным электрическое поле (которое генерирует ток I). Предположим, что носители заряда в материал — электроны, то электроны будут двигаться в направлении противоположный току (см. рисунок 31.7). Поскольку магнитное поле перпендикулярно электрическому полю, оно также перпендикулярно направлению движения электронов. В результате действия магнитной силы электроны отклоняются вниз, и на низ полоски. В то же время дефицит отрицательного заряда будет создаваться в верхней части полосы. Это распределение зарядов будет генерировать электрическое поле, перпендикулярное внешнему электрическому полю и под действием В условиях равновесия электрическая сила, создаваемая этим полем, уравновешивает магнитная сила, действующая на электроны.Когда это происходит, внутренний электрическое поле, E в , равно произведению электрона скорость v d и напряженность магнитного поля B. результатом внутреннего электрического поля будет создана разность потенциалов между верхом и низом полосы. Если металлическая полоса имеет ширину w, тогда разность потенциалов [Delta] V будет равна

(31,37)

Этот эффект называется эффектом Холла.

Скорость дрейфа электронов зависит от тока I в проводе, его площадь поперечного сечения A и плотность электронов n (см. главу 28):

(31,38)

Комбинируя уравнение (31.38) и уравнение (31.37), мы получаем следующее выражение для [Дельта] V

(31,39)

Следовательно, для определения n можно использовать измерение [Delta] V.

Ток I, протекающий по проводу, эквивалентен накоплению зарядов движется по проволоке с определенной скоростью v d .Количество заряд dq, присутствующий в отрезке dL провода, равен

(31,40)

Если провод помещен в магнитное поле, на него будет действовать магнитная сила. каждый из носителей заряда, и в результате на провод. Предположим угол между направлением тока и направлением поля равно [theta] (см. рисунок 31.8). Магнитная сила, действующая на отрезке dL провода равно

(31.41)

Полная сила, действующая на провод магнитным полем, может быть найдена следующим образом: интегрируя уравнение (31.41) по всей проволоке.

Рисунок 31.8. Магнитная сила на проводе.

Пример: Задача 31.33

Весы можно использовать для измерения силы магнитного поля. Рассмотрим петлю из провода, по которой проходит точно известный ток, как показано на рисунке. 31.9, который частично погружен в магнитное поле. Сила, которую магнитное поле, действующее на петлю, можно измерить с помощью весов, и это позволяет рассчитать напряженность магнитного поля.Предположим, что длина короткой стороны петли 10,0 см, сила тока в проводе 0,225 A, а магнитная сила составляет 5,35 x 10 -2 Н. Какова сила магнитное поле?

Рассмотрим три сегмента токовой петли, показанные на рисунке 31.9, которые погружен в магнитное поле. Магнитная сила, действующая на сегменты 1 и 3 имеют одинаковую величину, но направлены в противоположном направлении, и поэтому Отмена. Величина магнитной силы, действующей на сегмент 2, может быть рассчитывается с использованием ур.(31,41) и равно

(31,42)

Эта сила измеряется с помощью весов и равна 5,35 x 10 -2 . N. Таким образом, сила магнитного поля равна

. (31.43)

Рисунок 31.9. Токовая петля погружена в магнитное поле.

Если токовая петля погружена в магнитное поле, результирующая магнитная сила будет быть равным нулю. Однако крутящий момент на этом контуре, как правило, не будет равняется нулю.Предположим, прямоугольная токовая петля помещена в однородную магнитное поле (см. рисунок 31.10). Угол между нормалью тока петля и магнитное поле равно тета. Магнитные силы, действующие на верхняя и нижняя части токового контура равны

(31,44)

где L — длина верхнего и нижнего края. Крутящий момент, приложенный к токовая петля относительно ее оси равна

(31.45)

Рисунок 31.10. Токовая петля помещена в однородное магнитное поле. Используя определение магнитного дипольного момента u, обсуждаемое в главе 30 уравнение (31.45) можно переписать как

(31,46)

где

(31,47)

Используя векторные обозначения, уравнение (31.45) можно переписать как

(31,48)

где направление магнитного момента определяется с помощью правой правило.

Работа, которая должна выполняться против магнитного поля, чтобы вращать ток петля на угол d [theta] равна — [tau] d [theta]. Изменение в потенциальная энергия токового контура при его вращении между [theta] 0 и [theta] 1 равно

. (31,49)

Обычно в качестве опорной точки выбирается [theta] 0 = 90 градусов. а также U ([theta] 0 ) = 0 Дж. Если этот выбор сделан, мы можем переписать уравнение.(31,50) как

(31,50)

В векторной записи:

(31,51)

Потенциальная энергия токовой петли имеет минимум, когда u и B равны параллельно и максимум, когда u и B антипараллельны.


Отправляйте комментарии, вопросы и / или предложения по электронной почте на адрес [email protected] и / или посетите домашнюю страницу Фрэнка Вольфса.

22.2: Сила между двумя токоведущими проводами

Рассмотрим два бесконечных параллельных прямых провода на расстоянии \ (h \) друг от друга, по которым проходят восходящие токи, \ (I_ {1} \) и \ (I_ {2} \), соответственно, как показано на рисунке \ (\ PageIndex {1} \).

Рисунок \ (\ PageIndex {1} \): Два параллельных токоведущих провода будут оказывать притягивающее усилие друг на друга, если их токи идут в одном направлении.

Первый провод создаст магнитное поле \ (\ vec B_ {1} \) в форме кругов, концентрических по отношению к проводу. В положении второго провода магнитное поле \ (B_ {1} \) находится внутри страницы и имеет величину:

.

\ [\ begin {align} B_ {1} = \ frac {\ mu_ {0} I_ {1}} {2 \ pi h} \ end {align} \]

Поскольку второй провод проводит ток \ (I_ {2} \) вверх, он испытывает магнитную силу \ (\ vec F_ {2} \) от магнитного поля \ (B_ {1} \ ), то есть слева (как показано на рисунке \ (\ PageIndex {1} \) и определяется правилом правой руки).{◦} \). Мы ожидаем, исходя из Третьего закона Ньютона, что на первый провод должна действовать равная и противоположная сила. Действительно, второй провод создаст магнитное поле \ (\ vec B_ {2} \), которое находится за пределами страницы в месте расположения первого провода, с величиной:

.

\ [\ begin {align} B_ {2} = \ frac {\ mu_ {0} I_ {2}} {2 \ pi h} \ end {align} \]

Это приводит к магнитной силе \ (\ vec F_ {1} \), действующей на первый провод, который указывает вправо (от правила правой руки). На отрезке длины \ (l \) первого провода магнитная сила от магнитного поля \ (\ vec B_ {2} \) имеет величину:

.

\ [\ begin {align} F_ {1} = I_ {1} || \ vec l \ times \ vec B_ {2} || = I_ {1} lB_ {2} \ frac {\ mu_ {0} I_ {1} I_ {2}} {2 \ pi h} \ end {align} \]

, который действительно имеет ту же величину, что и сила, действующая на второй провод.Таким образом, когда два параллельных провода проводят ток в одном направлении, они оказывают друг на друга равные и противоположные силы притяжения.

Упражнение \ (\ PageIndex {1} \)

Рисунок \ (\ PageIndex {2} \): два провода, по которым ток проходит в противоположных направлениях.

По двум параллельным проводам ток проходит в противоположных направлениях, как показано на рисунке \ (\ PageIndex {2} \). Какую силу они оказывают друг на друга?

  1. Силы не будет, т. К. Токи отменяются.
  2. Между проводами будет сила притяжения.
  3. Между проводами будет сила отталкивания.
Ответ

Сила притяжения между двумя проводами служила основой для определения Ампера, базовой единицы измерения электрического тока. До 2019 года ампер определялся как «тот постоянный ток, который, если он поддерживается в двух прямых параллельных проводниках бесконечной длины, с незначительным круглым поперечным сечением и помещен на расстоянии одного метра друг от друга в вакууме, создавал бы между этими проводниками силу, равную \ (2 × 10 ^ {- 17} \ text {N} \) на метр длины ».{−19} \ text {C} \), а ампер соответствует одному кулону в секунду.

Сила между двумя проводами — хорошая система, чтобы понять, как никакая физическая величина не может зависеть от нашего выбора правой стороны для определения перекрестных произведений. Как упоминалось в предыдущей главе, любая физическая величина, такая как направление силы, действующей на провод, всегда будет зависеть от двух последовательных применений правой руки. В этой системе мы сначала использовали правило правой руки для аксиальных векторов, чтобы определить направление магнитного поля от одного из проводов.Затем мы использовали правило правой руки, чтобы определить направление перекрестного произведения, чтобы определить направление силы на другом проводе. Вы можете убедиться, что получите тот же ответ, если вместо этого воспользуетесь левой рукой, чтобы определить направление магнитного поля (которое будет в противоположном направлении), а затем снова для перекрестного произведения. Это также подчеркивает, что магнитное поле (и электрическое поле) — это просто математический инструмент, который мы используем, в конечном итоге, для описания движения зарядов или стрелок компаса.

Упражнение \ (\ PageIndex {2} \)

Когда по прямому кабелю течет ток, как можно ожидать, что заряды будут распределяться радиально по поперечному сечению кабеля?

  1. Равномерно по радиусу (плотность тока не зависит от \ (r \)).
  2. На внешней стороне кабеля будет избыток положительных зарядов.
  3. На внешней стороне кабеля будет избыток отрицательных зарядов.
Ответ

Магнитная сила на проводнике с током — University Physics Volume 2

Цели обучения

К концу этого раздела вы сможете:

  • Определите направление, в котором токоведущий провод испытывает силу во внешнем магнитном поле
  • Расчет силы на токоведущем проводе во внешнем магнитном поле

Движущиеся заряды испытывают силу в магнитном поле.Если эти движущиеся заряды находятся в проводе, то есть если по проводу проходит ток, на провод также должна действовать сила. Однако, прежде чем обсуждать силу, действующую на ток со стороны магнитного поля, мы сначала исследуем магнитное поле, создаваемое электрическим током. Здесь мы изучаем два отдельных эффекта, которые тесно взаимодействуют: провод с током создает магнитное поле, а магнитное поле оказывает силу на провод с током.

Магнитные поля, создаваемые электрическим током

Обсуждая исторические открытия в области магнетизма, мы упомянули открытие Эрстеда о том, что провод, по которому проходит электрический ток, вызывает отклонение расположенного поблизости компаса.Было установлено, что электрические токи создают магнитные поля. (Эта связь между электричеством и магнетизмом более подробно обсуждается в Источниках магнитных полей.)

Стрелка компаса рядом с проволокой испытывает силу, которая выравнивает касательную иглы к окружности вокруг проволоки. Следовательно, токоведущий провод создает кольцевые петли магнитного поля. Чтобы определить направление магнитного поля, создаваемого проводом, мы используем второе правило правой руки. В RHR-2 ваш большой палец указывает в направлении тока, в то время как ваши пальцы охватывают провод, указывая в направлении создаваемого магнитного поля ((Рисунок)).Если магнитное поле попадало на вас или выходило за пределы страницы, мы обозначаем это точкой. Если бы магнитное поле входило в страницу, мы представляем это с помощью этих символов. Эти символы появляются из рассмотрения векторной стрелки: стрелка, направленная на вас, с вашей точки зрения, будет выглядеть как точка или кончик стрелки. Стрелка, направленная от вас, с вашей точки зрения будет выглядеть как крест или составной эскиз магнитных кругов, показанный на (Рисунок), где показано, что напряженность поля уменьшается по мере удаления от провода петлями, которые дальше разделены.

(a) Когда проволока находится в плоскости бумаги, поле перпендикулярно бумаге. Обратите внимание на символы, используемые для поля, указывающего внутрь (например, хвоста стрелки), и поля, указывающего наружу (например, кончика стрелки). (б) Длинный и прямой провод создает поле с силовыми линиями магнитного поля, образующими кольцевые петли.

Расчет магнитной силы

Электрический ток — это упорядоченное движение заряда. Следовательно, провод с током в магнитном поле должен испытывать силу, создаваемую этим полем.Чтобы исследовать эту силу, давайте рассмотрим бесконечно малое сечение провода, как показано на (Рисунок). Длина и площадь поперечного сечения секции равны dl и A соответственно, поэтому ее объем равен. Проволока сформирована из материала, который содержит n носителей заряда на единицу объема, поэтому количество носителей заряда в сечение: Если носители заряда движутся со скоростью дрейфа, ток I в проводе равен (от Current and Resistance)

Магнитная сила на любом отдельном носителе заряда такова, что общая магнитная сила на носителях заряда в сечении провода составляет

Мы можем определить dl как вектор длиной dl , указывающий вдоль, что позволяет нам переписать это уравнение как

или

Это сила магнитного поля на отрезке провода.Обратите внимание, что на самом деле это результирующая сила, действующая со стороны поля на сами носители заряда. Направление этой силы задается RHR-1, где вы указываете пальцами в направлении тока и сгибаете их к полю. Затем ваш большой палец указывает в направлении силы.

Бесконечно малое сечение токоведущего провода в магнитном поле.

Чтобы определить магнитную силу на проводе произвольной длины и формы, мы должны интегрировать (рисунок) по всему проводу.Если сечение провода прямое, а B однородный, дифференциалы уравнения становятся абсолютными величинами, что дает нам

Это сила, действующая на прямой провод с током в однородном магнитном поле.

Уравновешивание гравитационных и магнитных сил на токоведущем проводе Провод длиной 50 см и массой 10 г подвешен в горизонтальной плоскости на паре гибких проводов ((Рисунок)). Затем на проволоку действует постоянное магнитное поле величиной 0.50 Т, который направлен, как показано. Каковы величина и направление тока в проводе, необходимые для снятия напряжения в опорных выводах?

(а) Проволока, подвешенная в магнитном поле. (б) Схема свободного тела для проволоки.

Стратегия Из диаграммы свободного тела на рисунке видно, что натяжения в опорных выводах стремятся к нулю, когда гравитационная и магнитная силы уравновешивают друг друга. Используя RHR-1, мы обнаруживаем, что магнитная сила направлена ​​вверх. Затем мы можем определить ток I , приравняв две силы.

Решение Приравняйте две силы веса и магнитного поля к проводу:

Таким образом,

Значение Это большое магнитное поле создает значительную силу на длине провода, чтобы противодействовать его весу.

Расчет магнитной силы на токоведущем проводе По длинному жесткому проводу, расположенному вдоль оси y , проходит ток 5,0 A, текущий в положительном направлении y . (a) Если постоянное магнитное поле величиной 0,30 Тл направлено вдоль положительной оси x , какова магнитная сила на единицу длины на проводе? (б) Если постоянное магнитное поле 0.30 T направлено на 30 градусов от оси + x к оси + y , какова магнитная сила на единицу длины на проводе?

Стратегия Магнитная сила, действующая на провод с током в магнитном поле, определяется по формуле а, поскольку в этой задаче ток и магнитное поле перпендикулярны, мы можем упростить формулу, чтобы получить величину и найти направление через RHR-1. Угол θ составляет 90 градусов, что означает, что длина также может быть разделена на левую часть, чтобы найти силу на единицу длины.Для части b текущая длина, умноженная на длину, записывается в обозначении единичного вектора, а также магнитное поле. После взятия перекрестного произведения направленность очевидна по результирующему единичному вектору.

Решение

  1. Начнем с общей формулы магнитной силы на проводе. Мы ищем силу на единицу длины, поэтому мы делим ее на длину, чтобы вывести ее в левую часть. Мы также устанавливаем Решение, следовательно,


    Направленность: Укажите пальцами в положительном направлении y и согните пальцы в положительном направлении x .Ваш большой палец укажет в направлении. Следовательно, с учетом направленности решение будет

  2. Текущее значение, умноженное на длину, и магнитное поле записываются в виде единичного вектора. Затем мы возьмем векторное произведение, чтобы найти силу:

Значение Это большое магнитное поле создает значительную силу на небольшой длине провода. По мере того, как угол магнитного поля становится более близким к току в проводе, на него действует меньшая сила, как видно из сравнения частей a и b.

Проверьте свое понимание Прямой гибкий медный провод погружается в магнитное поле, направленное внутрь страницы. (а) Если ток в проводе течет в направлении + x , в каком направлении будет изгибаться провод? (b) В какую сторону изгибается провод, если ток течет в направлении — x ?

а. наклоняется вверх; б. наклоняется вниз

Сила на круглом проводе Круговая токовая петля с радиусом R , по которой проходит ток I , расположена в плоскости xy .Постоянное однородное магнитное поле прорезает петлю параллельно оси y ((рисунок)). Найдите магнитную силу на верхней половине петли, нижней половине петли и общую силу на петле.

Петля из проволоки, по которой течет ток в магнитном поле.

Стратегия Магнитная сила на верхнем контуре должна быть записана в терминах дифференциальной силы, действующей на каждый сегмент контура. Если мы интегрируем по каждому дифференциальному элементу, мы решаем общую силу на этом участке петли.Сила, действующая на нижнюю петлю, определяется аналогичным образом, и общая сила складывается из этих двух сил.

Решение Дифференциальная сила на произвольном куске проволоки, расположенном на верхнем кольце, составляет:

где — угол между направлением магнитного поля (+ y ) и отрезком провода. Дифференциальный сегмент расположен на том же радиусе, поэтому, используя формулу длины дуги, мы имеем:

Чтобы найти силу на сегменте, мы интегрируем по верхней половине круга от 0 до. В результате получаем:

Нижняя половина цикла интегрирована от нуля до нуля, что дает нам:

Чистая сила — это сумма этих сил, которая равна нулю.

Значение Полная сила, действующая на любой замкнутый контур в однородном магнитном поле, равна нулю. Несмотря на то, что каждая часть петли имеет силу, действующую на нее, результирующая сила, действующая на систему, равна нулю. (Обратите внимание, что на петле есть чистый крутящий момент, который мы рассмотрим в следующем разделе.)

22.7 Магнитная сила на проводнике с током — Физика колледжа: OpenStax

Сводка

  • Опишите влияние магнитной силы на проводник с током.
  • Рассчитайте магнитную силу на проводнике с током.

Поскольку заряды обычно не могут покинуть проводник, магнитная сила, действующая на заряды, движущиеся в проводнике, передается на сам проводник.

Рис. 1. Магнитное поле воздействует на провод с током в направлении, заданном правилом правой руки 1 (в том же направлении, что и на отдельные движущиеся заряды). Эта сила может быть достаточно большой, чтобы переместить провод, поскольку типичные токи состоят из очень большого количества движущихся зарядов.

Мы можем получить выражение для магнитной силы, действующей на ток, суммируя магнитные силы, действующие на отдельные заряды. (Силы складываются, потому что они направлены в одном направлении.) Сила, действующая на отдельный заряд, движущийся со скоростью сноса vdvd, определяется как [latex] \ boldsymbol {F = qv_dB \; \ textbf {sin} \; \ theta} [ /латекс]. Если взять [латекс] \ boldsymbol {B} [/ latex], чтобы он был однородным по длине провода [латекс] \ boldsymbol {l} [/ latex] и ноль в другом месте, то общая магнитная сила на проводе будет [латекс] \ boldsymbol {F = (qv_dB \; \ textbf {sin} \; \ theta) (N)} [/ latex], где [latex] \ boldsymbol {N} [/ latex] — количество носителей заряда в секции проволока длиной [латекс] \ boldsymbol {l} [/ латекс].Теперь [латекс] \ boldsymbol {N = nV} [/ latex], где [латекс] \ boldsymbol {n} [/ latex] — количество носителей заряда в единице объема, а [латекс] \ boldsymbol {V} [/ латекс] — это объем провода в поле. Учитывая, что [латекс] \ boldsymbol {V = Al} [/ latex], где [латекс] \ boldsymbol {A} [/ latex] — это площадь поперечного сечения провода, то сила, действующая на провод, равна [латексу] \ boldsymbol {F = (qv_dB \; \ textbf {sin} \; \ theta) (nAl)} [/ latex]. Условия сбора,

[латекс] \ boldsymbol {F = (nqAv_d) lB \; \ textbf {sin} \; \ theta}.[/ латекс]

Поскольку [latex] \ boldsymbol {nqAv_d = I} [/ latex] (см. Главу 20.1 Current),

[латекс] \ boldsymbol {F = IlB \; \ textbf {sin} \; \ theta} [/ latex]

— это уравнение для магнитной силы , действующей на длину [латекс] \ boldsymbol {l} [/ latex] провода, по которому проходит ток [латекс] \ boldsymbol {I} [/ latex] в однородном магнитном поле [латекс] \ boldsymbol {B} [/ latex] , как показано на рисунке 2. Если мы разделим обе части этого выражения на [latex] \ boldsymbol {l} [/ latex], мы обнаружим, что магнитная сила на единицу длины провода в равномерное поле — это [латекс] \ boldsymbol {\ frac {F} {l} = IB \; \ textbf {sin} \; \ theta} [/ latex].Направление этой силы задается RHR-1 с большим пальцем в направлении текущего [латекса] \ boldsymbol {I} [/ latex]. Затем, пальцы в направлении [латекс] \ boldsymbol {B} [/ latex], перпендикуляр к ладони указывает в направлении [латекс] \ boldsymbol {F} [/ латекс], как на рисунке 2.

Рис. 2. Сила, действующая на токоведущий провод в магнитном поле, составляет F = IlB sin θ . Его направление задает RHR-1.

Расчет магнитной силы на проводе с током: сильное магнитное поле

Вычислите силу, действующую на провод, показанную на Рисунке 1, при условии [латекс] \ boldsymbol {B = 1.{\ circ}} [/ latex], так что [latex] \ boldsymbol {\ textbf {sin} \; \ theta = 1} [/ latex].

Раствор

Ввод заданных значений в [latex] \ boldsymbol {F = IlB \; \ textbf {sin} \ theta} [/ latex] дает

[латекс] \ boldsymbol {F = IlB \; \ textbf {sin} \ theta = (20.0 \; \ textbf {A}) \; (0,0500 \; \ textbf {m}) \; (1.50 \; \ textbf {T}) \; (1)}. [/ Latex]

Единицы измерения теслы: [латекс] \ boldsymbol {1 \; \ textbf {T} = \ frac {\ textbf {N}} {\ textbf {A} \ cdot \; \ textbf {m}}} [/ латекс]; таким образом,

[латекс] \ boldsymbol {F = 1.50 \; \ textbf {N}}. [/ Latex]

Обсуждение

Это большое магнитное поле создает значительную силу на небольшой длине провода.

Магнитная сила на токоведущих проводниках используется для преобразования электрической энергии в работу. (Двигатели являются ярким примером — они используют проволочные петли и рассматриваются в следующем разделе.) Магнитогидродинамика (MHD) — это техническое название, данное умному приложению, в котором магнитная сила перекачивает жидкости без движущихся механических частей.(См. Рисунок 3.)

Рисунок 3. Магнитогидродинамика. Магнитная сила, действующая на ток, проходящий через эту жидкость, может использоваться в качестве немеханического насоса.

К трубке прикладывается сильное магнитное поле, и через жидкость проходит ток под прямым углом к ​​полю, в результате чего сила, действующая на жидкость, параллельна оси трубки, как показано. Отсутствие движущихся частей делает его привлекательным для перемещения горячего химически активного вещества, такого как жидкий натрий, используемый в некоторых ядерных реакторах.Экспериментальные искусственные сердца проходят испытания с использованием этого метода перекачивания крови, возможно, чтобы избежать неблагоприятного воздействия механических насосов. (Однако на клеточные мембраны влияют большие поля, необходимые в МГД, что задерживает его практическое применение у людей.) МГД-двигательная установка для атомных подводных лодок была предложена, поскольку она могла бы быть значительно тише, чем обычные гребные винты. Сдерживающая ценность атомных подводных лодок основана на их способности укрыться и пережить первый или второй ядерный удар.По мере того как мы медленно разбираем наши арсеналы ядерного оружия, подразделение подводных лодок будет выводиться из эксплуатации последним из-за этой способности (см. Рис. 4.) Существующие приводы MHD тяжелы и неэффективны — требуется большая работа по развитию.

Рис. 4. Двигательная установка MHD на атомной подводной лодке может создавать значительно меньшую турбулентность, чем гребные винты, и позволять ей работать более бесшумно. Создание подводной лодки с бесшумным двигателем было инсценировано в книге и фильме «Охота за красным октябрем ».
  • Магнитная сила на токоведущих проводниках определяется выражением

    [латекс] \ boldsymbol {F = IlB \; \ textbf {sin} \; \ theta}, [/ latex]

    где [латекс] \ boldsymbol {I} [/ latex] — ток, [латекс] \ boldsymbol {l} [/ latex] — длина прямого проводника в однородном магнитном поле [латекс] \ boldsymbol {B} [/ latex], а [latex] \ boldsymbol {\ theta} [/ latex] — это угол между [latex] \ boldsymbol {I} [/ latex] и [latex] \ boldsymbol {B} [/ latex]. Сила следует за RHR-1 большим пальцем в направлении [latex] \ boldsymbol {I} [/ latex].

Концептуальные вопросы

1: Нарисуйте схему ситуации на рисунке 1, показывающую направление электронов, переносящих ток, и используйте RHR-1, чтобы проверить направление силы на провод.

2: Убедитесь, что направление силы в приводе MHD, таком как на рисунке 3, не зависит от знака зарядов, переносящих ток через жидкость.

3: Почему магнитогидродинамический привод лучше работает в океанской воде, чем в пресной? Кроме того, зачем нужны сверхпроводящие магниты?

4: Что с большей вероятностью повлияет на показания компаса, переменный ток в холодильнике или постоянный ток при запуске автомобиля? Объяснять.

Задачи и упражнения

1: Какое направление магнитной силы действует на ток в каждом из шести случаев на рисунке 5?

Рис. 5.

2: Каково направление тока, который испытывает магнитную силу, показанную в каждом из трех случаев на рис. 6, при условии, что ток течет перпендикулярно [латексу] \ boldsymbol {B} [/ latex ]?

Рисунок 6

3: Каково направление магнитного поля, которое создает магнитную силу, показанную на токах в каждом из трех случаев на рисунке 7, при условии, что [latex] \ boldsymbol {B} [/ latex] перпендикулярно [латексу] \ boldsymbol {I} [/ latex]?

Рисунок 7.{-5} — поле \ textbf {T}} [/ latex]. Какая сила действует на 100-метровом участке этой линии? (b) Обсудите практические проблемы, которые это представляет, если таковые имеются.

6: Какая сила действует на воду в МГД-приводе, использующем трубку диаметром 25,0 см, если через трубку проходит ток 100 А, перпендикулярный магнитному полю 2,00 Тл? (Относительно небольшой размер этой силы указывает на необходимость очень больших токов и магнитных полей для создания практических МГД-приводов.)

7: Провод, несущий 30.{\ circ}} [/ latex] с полем?

10: Сила, действующая на прямоугольную петлю провода в магнитном поле на Рисунке 8, может использоваться для измерения напряженности поля. Поле однородное, плоскость петли перпендикулярна полю. а) Каково направление магнитной силы на петле? Обоснуйте утверждение, что силы на сторонах петли равны и противоположны, независимо от того, какая часть петли находится в поле, и не влияют на результирующую силу, действующую на петлю. (b) Если ток 5.00 A, какова сила на тесла на петле шириной 20,0 см?

Рисунок 8.

Решения

Задачи и упражнения

1: (а) запад (слева)

(b) на стр.

(в) север (верх)

(d) нет силы

(д) восток (правый)

(е) юг (низ)

3: (a) на стр.

(б) запад (слева)

(c) вне страницы

5: (а) 2.{\ circ}} [/ латекс]

(б) 4.80 с.ш.

экспериментов Ампера

экспериментов Ампера
Далее: Закон Ампера Up: Магнетизм Предыдущая: Историческая справка


Эксперименты Ампера В 1820 году датский физик Ганс Кристиан Эрстед читал лекцию. демонстрация различных электрических и магнитные эффекты. Вдруг, к своему изумлению, он заметил, что стрелка компаса, которую он держал отклонился, когда он поднес его к токопроводящей провод.Это было очень удивительное наблюдение, поскольку до этого момента электричество и магнетизм считался двумя совершенно не связанными друг с другом явлениями. Слух об этом открытии быстро распространился по научной лозе. и французский физик Андре Мари Ампер сразу решил разобраться дальше. Аппарат Ампера состоял (по существу) из длинного прямого провода, несущего электрический ток Текущий . Ампер быстро обнаружил, что стрелка небольшого компаса отображает из ряда концентрических круговых петель в плоскости перпендикулярно токоведущему проводу — см. рис.20. Направление циркуляции вокруг этих магнитных петель принято считать направление, в котором север полюс стрелки компаса точки. Используя это соглашение, циркуляция петель задается линейка правая . Если большой палец правой руки указывает в направлении тока, то пальцы правой руки вращаются в том же смысле, что и магнитные петли.
Рисунок 20: Магнитные петли вокруг токоведущего провода.

Следующая серия экспериментов Ампера заключалась в том, чтобы принести короткий испытательный провод, ток, рядом с исходным проводом и исследуя силу, приложенную к испытательному проводу. Этот эксперимент не так ясен, как эксперимент Кулона, потому что, в отличие от электрические заряды, электрические токи не могут существовать как точечные сущности. Они должны течь по замкнутым контурам. Мы должны представьте, что цепь, которая соединяется с центральным проводом, достаточно далеко, что это не оказывает заметного влияния на результат эксперимента.Цепь, которая соединяется с тестовый провод более проблематичен. К счастью, если питающие проволоки скручены друг вокруг друга, как показано на рис.21, затем они эффективно нейтрализуют друг друга, а также не влияют на результат эксперимент.

Рисунок 21: Эксперимент Ампера.

Ampère обнаружил, что сила, действующая на испытательный провод, прямо пропорциональна к его длине.Он также сделал следующие наблюдения. Если ток в тестовом проводе (, т.е. , испытательный ток) течет параллельно току в центральном проводе. тогда два провода притягиваются друг к другу. Если ток в тесте провод перевернут, тогда два провода отталкиваются друг от друга. Если испытательный ток направлен радиально к центральному проводу (и ток в центральном проводе течет вверх), затем тестовый провод подвергается действию направленной вниз силы. Если испытательный ток поменять местами, сила равна вверх.Если испытательный ток вращается в одной плоскости, так что он начинается параллельно центральному току и заканчивается радиально к нему, затем сила на испытательный провод имеет постоянную величину и всегда находится под прямым углом к испытательный ток. Если испытательный ток параллелен магнитной петле, то есть к испытательному проводу не прикладывалась сила. Если испытательный ток вращается в одна плоскость, так что она начинается параллельно центральному току и заканчивается указывая вдоль магнитной петли, то величина силы, действующей на тестовый провод затухает как (где угол тока перевернут, и соответствует случай, когда испытательный ток параллелен центральному току), и его направление снова всегда под прямым углом к тестовый ток.Наконец, Ампер смог установить, что привлекательный сила между двумя параллельными токоведущими проводами пропорциональна произведению два тока, и падает как перпендикуляр расстояние между проводами.

Этот довольно сложный силовой закон можно кратко описать в векторных обозначениях. при условии, что мы определяем векторное поле, называемое магнитным полем , который заполняет пространство, и направление которого всюду касательно магнитные петли, нанесенные на карту севером полюс малого компас.Зависимость силы на единицу длины, действующей на испытательный провод с разными возможные направления испытательного тока описываются

(152)

где — вектор, направление и величина которого такие же, как у испытательного тока.

Изменение силы на единицу длины, действующей на испытательный провод с прочностью центральный ток и расстояние по перпендикуляру к центральному проводу объясняется тем, что напряженность магнитного поля пропорциональна, и обратно пропорционально.Таким образом, мы можем написать

(153)

Константа пропорциональности называется проницаемость свободного пространства , и принимает значение
(154)

Между прочим, единицей измерения напряженности магнитного поля в системе СИ является тесла (Тл), т.е. то же, что и ньютон на ампер на метр:
(155)

Понятие о магнитном поле, которое заполняет пространство вокруг токоведущего провода позволяет рассчитать силу при испытании провод удобно разделить на две части.В первой части мы вычисляем магнитное поле, создаваемое током, протекающим в центральном проводе. Это поле циркулирует в плоскости, перпендикулярной проводу. Его величина составляет пропорциональна центральному току и обратно пропорциональна перпендикуляру расстояние от провода. Во второй части мы используем Уравнение (152) для расчета силы на единицу длина, действующая на короткий токопроводящий провод, помещенный в магнитное поле генерируется центральным током. Эта сила перпендикулярна как направлению магнитного поля, так и направлению магнитного поля. испытательный ток.Отметим, что на данном этапе у нас нет оснований предполагать, что магнитная поле реально существует. Он введен просто для облегчения расчета. силы, приложенной к испытательному проводу центральным проводом. Оказывается, однако, что магнитное поле действительно существует, поскольку, как мы увидим, существует энергия, связанная с магнитным полем, заполняющим пространство.



Далее: Закон Ампера Up: Магнетизм Предыдущая: Историческая справка
Ричард Фицпатрик 2007-07-14

Общая физика II

Источники магнитного поля

Вопросы 1, 2, 3, 4, 9, 18, 19

Проблемы 30.8А, 22, 23, 25, 32, 40, 60, 64


Q1 Магнитное поле создается токовой петлей? униформа?

Вовсе нет; он меняется в зависимости от положения — он меняется в направлении и величина — как показано на этом скетче:

Q2 Ток в проводнике создает магнитное поле который можно рассчитать по закону Био-Савара. Поскольку ток определяется как скорость потока заряда, что вы можете сделать вывод о магическое поле, создаваемое стационарным зарядом? Что о движущиеся обвинения?

Магнитное поле, создаваемое стационарным зарядом , равно ноль. Движущийся заряд равен току, поэтому будет производят магнитного поля.

Q3 Два параллельных провода несут токи друг напротив друга. направления. Опишите природу результирующего магнитного поля. создается двумя проводами в точках

(а) между проводами и

или

Между проводами магнитные поля от двух провода указывают в направлении в том же направлении , поэтому результирующий или общий магнитное поле будет большим.

(б) вне проводов в плоскости, содержащей их.

или

За пределами проводов два магнитных поля от двух токов в противоположных направлениях, поэтому они будут иметь тенденцию отменять друг друга и результирующее или полное магнитное поле будет малым .

Q4 Объясните, почему по двум параллельным проводам проходят токи в противоположных направлений отталкиваются друг от друга.

Вы можете посмотреть диаграммы для Вопрос 3 . В В частности, рассмотрим эту диаграмму,

или

Магнитное поле левого тока вызывает силу в правом ток, который является отталкиванием, поэтому мы знаем — из Третьего закона Ньютона — что два тока отталкиваются друг от друга.

Q9 Опишите сходство между законом Ампера в магнетизм и закон Гаусса в электростатике.

Оба связывают интеграл вокруг замкнутой поверхности или линии с что-то, что проходит через поверхность или линию интеграла. Закон Гаусса гласит, что электрический поток через замкнутую поверхность равен пропорционально заряду на этой поверхности. Закон Ампера гласит линейный интеграл B -d s вокруг замкнутого пути равен пропорционально току, который проходит по этому пути.

Q18 Притягивается ли гвоздь к любому полюсу магнита? Объясните, что происходит внутри гвоздя, когда его кладут рядом с магнит.

Да, гвоздь будет притягиваться к любой полюс магнит. Магнит заставляет границы между магнитными доменами слегка двигаться, создавая чистое магнитное поле для гвоздя. path Если гвоздь находится рядом с другим магнитным полюсом, а затем с другим магнитным полюсом границы домена перемещаются.

Q19 Северный полюс магнита притягивается. к географическому Северному полюсу Земли. Тем не менее, полюсы вроде пополняются. Что здесь происходит?

Магнитный полюс около географического Северного полюса Земли — это то, что иначе назвали бы магнитным полюсом , направленным на юг.


30,8 А, Проводник состоит из круглой петли радиуса R и две прямые длинные секции, как на рисунке P30.8. Провод лежит в плоскости бумаги и несет ток I. Определить величина и направление магнитного поля в центре петля.

Мы знаем, как справиться с магнитным полем благодаря прямой провод;

На расстоянии R от провода магнитное поле равно

И мы знаем, как обращаться с магнитным полем в центре круговая петля;

В центре провода магнитное поле

Итак, наше полное магнитное поле в центре круговой петли а расстояние R от длинного прямого провода равно сумме эти,

30.22, Длинный соленоид общей длины с плотной обмоткой 30,0 см (0,30 м) имеет магнитное поле 4,00 · 10-4 Тл на своем центр, создаваемый током 1,00 А через его обмотки. Как много витков провода на соленоиде?

Магнитное поле внутри соленоида определяется выражением

или

п = В / я

n = (4 x 10 — 4 ) / [(4 x 10 — 7 ) (1)]

n = 318 витков / м

п = н / л

N = n L

N = (318 витков / м) (0.30 м)

N = 95 витков

30,23, Сверхпроводящий соленоид должен генерировать магнитное поле 10,0 Тл.

(a) Если обмотка соленоида имеет 2000 витков / м, какой требуемый ток?

или

I = B / п

I = (10) / [(4 х 10 — 7 ) (2000)]

I = 3979 А

Это сильнейшее течение; но тогда это огромный магнитное поле!

30.25, Некоторые сверхпроводящие сплавы при очень низких температуры могут переносить очень высокие токи. Например Nb 3 Sn-провод при 10 К выдерживает ток 1000 А и сохраняет свою сверхпроводимость. Определите максимальное значение B, достижимое в соленоид длиной 25 см (0,25 м), если 1000 оборотов Nb 3 Sn намотаны на внешнюю поверхность.

B = (4 х 10 — 7 ) (1000 / 0,25) (1000)

B = 5 т

30.32, На рисунке P30.32, оба тока находятся в отрицательное направление x.

(a) Изобразите картину магнитного поля в плоскости yz.

Помните, это набросок !

30,40, Рассмотрим полусферическую замкнутую поверхность на рисунке P30.40. Если полушарие находится в однородном магнитном поле, которое создает угол тета по вертикали рассчитать магнитный поток через

(а) плоская поверхность S 1 и

= A B cos

A = 2

знак равно R 2 B cos

(б) полусферическая поверхность S 2 .

Согласно закону Гаусса для магнитного потока общий магнитный поток равен ноль . Поскольку мы знаем поток приход на поверхность ( знак равно R 2 B cos) это (или отрицательное ) из этого также является потоком, идущим из поверхности — поток через S 2 ,

знак равно R 2 B cos

30,60, Молния может пропускать ток 10 000 А на короткий период времени.Какое получается магнитное поле 100 м от ботл?

B = [(4 x 10 — 7 ) (10 000)] / [(2) (100)]

B = 2 x 10 — 5 T

30,64 Два параллельных проводника проводят ток в противоположных направлениях. направления, как показано на рисунке P30.64. Один проводник несет ток 10А. Точка А находится посередине между проводами и точка C находится на расстоянии d / 2 вправо от тока 10 А. Если d = 18 см = 0.18 м и I настроен так, чтобы магнитное поле в точке C было ноль, найди

(а) значение тока I и

(б) значение магнитного поля при A.

для пункта C ,

B справа = [(4 x 10 — 7 ) (10)] / [(2) (0,09)] = 2,22 x 10 — 5 т

B слева = [(4 x 10 — 7 ) ( I )] / [(2) (0,27)] = 2,22 x 10 — 5 т

I = (2.22 x 10 — 5 ) (0,27) / 2 x 10 — 7

I = 30 A

для пункта A ,

B справа = [(4 x 10 — 7 ) (10)] / [(2) (0,09)] = 2,22 x 10 — 5 т

B слева = [(4 x 10 — 7 ) (30)] / [(2) (0,09)] = 6,66 x 10 — 5 T

Используя Правило правой руки, мы видим, что эти два поля указывают в том же направлении — «вверх» или «со страницы» в точке А

B всего = B справа + B слева = 8.88 x 10 — 5 т c) Дуг Дэвис, 2002 год; все права защищены

Электромагнитные силы и поля

Стержневой магнит притягивает к своим концам железные предметы, называемые полюсами . Один конец — это северный полюс , а другой — южный полюс . Если стержень подвешен так, чтобы он мог свободно двигаться, магнит выровняется так, чтобы его северный полюс указывал на географический север Земли. Подвешенный стержневой магнит действует как компас в магнитном поле Земли.Если два стержневых магнита приблизить друг к другу, одинаковые полюса будут отталкиваться друг от друга, а разные полюса притянутся друг к другу. (Примечание: согласно этому определению, магнитный полюс под северным географическим полюсом Земли является южным полюсом магнитного поля Земли.)

Это магнитное притяжение или отталкивание можно объяснить как влияние одного магнита на другой, либо оно может Можно сказать, что один магнит создает в области вокруг себя магнитное поле , которое воздействует на другой магнит.Магнитное поле в любой точке — это вектор. Направление магнитного поля ( B ) в указанной точке — это направление, в котором северный конец стрелки компаса указывает в этом положении. Линии магнитного поля , аналогичные силовым линиям электрического поля, описывают силу, действующую на магнитные частицы, находящиеся внутри поля. Железные опилки будут выровнены, чтобы обозначить структуру силовых линий магнитного поля.

Сила движущегося заряда

Если заряд движется через магнитное поле под углом, он испытывает силу.Уравнение задается следующим образом: F = q v × B или F = qvB sin θ, где q — заряд, B — магнитное поле, v — скорость и θ — угол между направлениями магнитного поля и скорости; таким образом, используя определение перекрестного произведения, определение магнитного поля равно

Магнитное поле выражается в единицах СИ как тесла (Тл), которую также называют Вебером на квадратный метр:

Направление F определяется по правилу правой руки, показанному на рисунке 1.

Рисунок 1

Используя правило правой руки, найдите направление магнитной силы на движущийся заряд.

Чтобы найти направление силы, действующей на заряд, плоской рукой направьте большой палец в направлении скорости положительного заряда, а пальцы — в направлении магнитного поля.Направление силы вне ладони. (Если движущийся заряд отрицательный, укажите большим пальцем в направлении, противоположном его направлению движения.) Математически эта сила является перекрестным произведением вектора скорости и вектора магнитного поля.

Если скорость заряженной частицы перпендикулярна однородному магнитному полю, сила всегда будет направлена ​​к центру круга радиусом r , как показано на рисунке 2. x символизирует магнитное поле в плоскость бумаги — хвост стрелки.(Точка символизирует вектор, выходящий из плоскости бумаги — кончик стрелки.)

Рисунок 2

Сила, действующая на заряд, движущийся перпендикулярно магнитному полю, направлена ​​к центру круга.

Магнитная сила обеспечивает центростремительное ускорение:

или

Радиус пути пропорционален массе заряда.Это уравнение лежит в основе работы масс-спектрометра , который может разделять одинаково ионизированные атомы немного разных масс. Однократно ионизированным атомам придаются равные скорости, и поскольку их заряды одинаковы и они проходят через один и тот же B , они будут двигаться немного разными путями и затем могут быть разделены.

Сила на токоведущем

Заряды, удерживаемые в проводах, также могут испытывать силу в магнитном поле.Ток (I) в магнитном поле ( B ) испытывает силу ( F ), заданную уравнением F = I l × B или F = IlB sin θ, где l — длина провода, представленная вектором, указывающим в направлении тока. Направление силы можно определить по правилу правой руки, аналогичному показанному на рисунке. В этом случае направьте большой палец в направлении тока — направлении движения положительных зарядов.Ток не будет испытывать силы, если он параллелен магнитному полю.

Крутящий момент в токовой петле

Цепь тока в магнитном поле может испытывать крутящий момент, если она свободно вращается. На рисунке (а) изображена квадратная петля из проволоки в магнитном поле, направленном вправо. Представьте на рисунке (b), что ось провода повернута на угол (θ) с магнитным полем, и что вид смотрит вниз на верхнюю часть петли.

Author: alexxlab

Добавить комментарий

Ваш адрес email не будет опубликован. Обязательные поля помечены *